ELECTROMECANICA Y MAQUINAS ELECTRICAS

January 9, 2018 | Author: quique | Category: Electric Power, Electricity, Transformer, Design, Physics & Mathematics
Share Embed Donate


Short Description

Descripción: ELECTROMECANICA Y MAQUINAS ELECTRICAS...

Description

ELECTROMECANICA y MAQUINAS ELECTRICAS

ELECTROMECANICA YMAQUINAS /

ELECTRICAS

S.A.

Nasal

Departamento de Ingeniería Eléctríca Universidad de Kentucky Lexington, Kentucky

L.E. Unnewehr Laboratorios de InvesNgación Cientmca Ford Motor Company Dearborn¡ Michigan

.

..

LIMUSA

VelSión autorizada en eapatiol de la obra publJeadaea l.ngl& por Jolln Wüey & Sons bajo el titulo do ELECIROMECHANICS ANO ELECnUC MACHINES O 1979 by Jolln Wüey &; Sona, Inc. ISBN 6471-GJ536-X Vonión espalo": GUILLERMO GARCIA T ALAVERA

lnpJüero en ComwdcacioneJ y Electrónica y ex·Profesor de Teoría de Jos Circuitos

E1~. en la &;ueIa Superior de lD&ui«ía Mecánica y EJéctrica del lrutttuto Politécnico Nacional de México

Revisión: . VK70RPEREZAMADORBARRON lJI¡eIüero Mecánico E1ectr.iáata. Profesor TItular de Tiempo Completo y Jefe de la Secci6n de IQeemcJía de la Universidad Nacional Autónoma de Méxil:o. Profesor del Departamento de FÚlica de la Universidad lberoamtll'ial.lla

Todos Joa derechos luenados: @ 19' 7 EDITORIAL LIMUSA. S.A.

Bateleras 95. Primer piso, México 1, D.F'. Miembro de la Camara Nacional de la Industria EditorlaI. Rq¡istro Núm. 111

Pdm_ edidón: 1982 lmpHlO en Méxko (2819)

ISBN .....'8-1362-6

A mi tío el profesor Majnoon Gorakhpuri, quien con sus inconmensurables aportaciones literarias influyó en mI: Con el más sincero agradecimiento y afecto, S.A. Nasar A lean, por su amor y comprensión, L.E. Unnewher

I

I I I I I I I

I I I I I I I

I I I I I I I I I I I I I

Prólogo

ra el hogar y la provisión de la mayor parte del trabajo mecánico en la industria, proporcionar un importante proceso de conversión de en la producción de gía eléctrica. Estos dispositivos electromecánicos también 4,:~il~i)eñáir UlÍli.( fl11flé~i~p

COlmplonlentes en ~.~'.v.....~ visual. En resumen, los dispositivos electromecánicos forman parte de la mayoría de los sistemas con los que deben trabajar los ingenieros y tecnólogos de muchas "",,,u,unas. Es por esto que es tan importante conocer los v0:1taje in:dliGUIó y,'qampos'clect se muestra a lo largo de la longitud media del miembro magnético y a través del entrehierro de aire. La línea de integración se recorrerá en el sentido de las manecillas del reloj. En las tres bobinas se indican las direcciones de corriente. Obsérvese que para las direcciones indicadas, la dirección de corriente penetra al plano del papel para los conductores incluidos por la trayectoria de integración en las bobinas 1 y 3 y sale del plano del papel para la 2. Del primer miembro de la ecuación (2.2) se obtiene (2.l9)

Si el material magnético es lineal, hom'Jgéneo e isotrópico y si no se toma en cuenta el flujo de fuga, la ecuación (2.19) se convierte en (2.20)

donde Rm y Rg son las reluctancias del miembro magnético y del entrehierro, respectivamente y, Mm y Mg representan los potenciales magnéticos o caida de reluctancia a través de estos dos elementos del circuito magnético. El segundo miembro de la ecuación (2.2) da (2.21)

La combinación de la ecuaciones (2.20) y (2.21) proporciona (2.22)

Se puede generalizar la ley de Ampére en base a este sencillo ejemplo estableciendo que "la suma de los potenciales magnéticos alrededor de cualquier

Ley de Ampére aplicada a un circuito magnético 55

Núcleo de hierro

/,

'+--+--+- N,

FIGURA 2.8 Circuito magnético compuesto, con excitación múltiple (fmms)

trayectoria cerrada es igual a cero", análoga a la relación de voltaje de Kirchhoff en los circuitos eléctricos. Procede notar que esta generalización se infiere aún sin los aspectos de simplificación usados para eliminar la forma integral de la ecuación (2.19). Ejemplo 2.1

Ilustremos el uso de la ecuación (2.22) dando valores numéricos para el circuito de la figura 2.8 y resolviendo el siguiente problema: determinar el número de amperio-vueltas requerido para establecer una densidad de flujo de un tesla en el entrehierro de aire. Aquí no tiene caso incluir las tres bobinas, así que se cancelarán 12 e 13 , y se buscará el producto l I N I • Sea 0.1 mm la longitud del entrehierro de aire. El miembro magnético se supone construido de acero laminado M-19 con factor de pila de 0.9 y una longitud lm de 100 mm; no se tomarán en cuenta las dispersiones de borde y. de fuga. Las caídas de reluctancia pueden calcularse mediante cualquiera de los miembros de la ecuación (2.20). Dado que la densidad de flujo en el entrehierro de aire es un dato, el cálculo de la intensidad de campo magnético es simple: B H = --.!.. = 1.0 =7.95x 105 A/m g /lo 4'17 X 10- 7

Si no se toman en cuenta las dispersiones de fuga y de borde, la densidad de flujo en el miembro magnético puede estimarse que es igual a la del entrehierro de aire, dividida entre el factor de pila:

56 Circuitos magnéticos de CA. y C. C. estacionarios

De la curva para el acero M-19 en la figura 2.4, en este valor de densidad de flujo H=130

Mm = 130xO.l = 13

El número de amperio-vueltas requerido en la bobina de excitación N/ 1 = Mm + Mg =92.5 A (amperio-vuelta) Ejemplo 2.2.

Para la misma configuración y los mismos valores numéricos, determinar los amperio-vuelta requeridos en la bobina de excitación para establecer un flujo de 0.001 Wb en el entrehierro de aire. El problema se resolverá mediante reluctancias, sin tomar en cuenta las pérdidas por fuga, pero sí los efectos de borde. Es preciso conocer el área de la sección transversal del miembro magnético para determinar las reluctancias; se considerará que Am = 16 cm 2 aproximadamente. Para determinar la reluctancia del entrehierro de aire se puede usar la ecuación (2.18). Supóngase que los efectos de borde aumentan el áreaefectiva del entrehierro en un 10% respecto al área de la superficie del acero que lo limita. La reluctancia es entonces R = g

10-

4

(4'1TX 10- 7 )(1.1 X 0.0016)

=4.5 X 104

Al no tomar en cuenta las fugas, existe el mismo flujo en todo el miembro magnético. La densidad de flujo en el material magnético es Bm =

0.9~~~16 =0.695 tes la

De la curva M-19 de la figura 2.4, la permeabilidad en amplitud es

La reluctancia del miembro magnético es R = m

1m = 0.1 =0.54 X 104 JLRJLoAm 10240(4'1TXIO- 7 )(0.9xO.0016)

Limitaciones del método del circuito magnético 57

FIGURA 2.9 Circuito equivalente aproximado para la figura 2.8.

Los amperio-vuelta de excitación requeridos son

Hay varias conclusiones que pueden observarse de estos ejemplos sencillos:

1. El núcleo magnético con entrehierro de aire es análogo a un circuito serie simple de C.C., como se muestra en la figura 2.9. 2. Debido a la simetría respecto al plano del papel del sistema de la figura 2.8, es aceptable la representación bidimensional del campo magnético. 3. El cálculo de reluctancias es un método más complejo que el uso de intensidades de campo magnético para determinar las caídas de reluctancia. 4. Probablemente, se dudará tratar de obtener la solución de los problemas inversos de los dos ejemplos anteriores; es decir, dado el número de amperio-vueltas de excitación, determinar el flujo (o densidad de flujo) en el entrehierro de aire. Si se reflexiona un poco, se observará que no existe una solución analítica directa a este problema debido a la no linealidad de la característica B-H del material magnptico para acceso repetido durante el proceso de iteración. 2.6 LIMITACIONES DEL METODO DEL 1..,RCUITO MAGNETICO

La cantidad de problemas de circuitos magnéticos prácticos que pueden resolverse mediante el método expuesto en las dos secciones precedentes es más bien limitada, a pesar de la semejanza de este método con la teoría elemental de los circuitos eléctricos de c.c. La discusión (4) subsiguiente al ejemplo 2.2 ilustró sólo una de las limitaciones. El objetivo de introducir circuitos magnéticos es más con la intención de establecer algunos principios y definiciones fundamentales, que como técnica para resolver problemas. Las limitaciones de la teoría de los circuitos magnéticos se derivan principalmente en la naturaleza de los materiales magnéticos, que difiere ostensiblemente de la de los materiales conductores, aislantes y dieléctricos. La mayor parte de estas limitaciones se había ya incluido en cali-

58 Circuitos magnéticos de CA. y C. C. estacionarios

dad de "suposiciones" y se expuso en la discusión acerca de los circuitos magnéticos. Es necesario precisar el significado de estas suposiciones:

1. Material magnético homogéneo: la mayoría de los materiales usados en sistemas electromagnéticos prácticos pueden considerarse homogéneos dentro de regiones finitas del espacio, permitiendo el uso de las formas integrales de las ecuaciones de Maxwell y los cálculos de reluctancias y permeancias. 2. Materiales magnéticos isotrópicos; casi todos los aceros en láminas y las ferritas se orientan por medio de procesos metalúrgicos durante su producción. Los materiales orientados tiene una dirección "favorecida" en su estructura granular, dando propiedades magnéticas superiores al ser magnetizados a lo largo de esta dirección. Cuando existan estas características direccionales, siempre deben tomarse en cuenta al analizar el circuito magnético, aunque en general, las ecuaciones integrales aún son aplicables. 3. Característica no lineal; ésta es una propiedad inherente a todos los materiales ferro y ferrimagnéticos, cosa que se puede tomar como un obstáculo serio para cualquier método analítico que se intente en sistemas electromagnéticos, pero hay varias razones para aseverar que no es así: a) Como se puede apreciar en las curvas B-H expuestas en las figuras

2.1 y 2.4 hay una vasta sección de la curva, con la mayoría de los materiales, que se puede aproximar a una línea recta. Muchos dispositivos se operan en densidades de flujo principalmente en este intervalo y el dispositivo se puede considerar, en esencia lineal. b) La característica B-H no lineal de los materiales magneticos se manifiesta en la relación entre flujo y corriente de excitación en sistemas electromagnéticos. La relación entre flujo y voltaje inducido es una relación lineal, dada por la ley de Faraday, ecuación (2.3). Es posible tratar por separado estas características de excitación no lineales en muchos sistemas, tal como se hace en el método del circuito equivalente para transformadores y motores de inducción. e) Una inductancia cuyo circuito magnético esté constituido por un material magnético, es un elemento no lineal de un circuito eléctrico, como una bobina devanada en un toroide magnético. Sin embargo, con un entrehierro de aire en el toroide magnético, se reduce el efecto del material magnético no lineal sobre la inductancia. Las máquinas rotatorias y muchos otros dispositivos electromecánicos tienen entehierros de aire en sus circuitos magnéticos, permitiendo que la teoría básica de estos dispositivos se describa mediante ecuaciones lineales. Como se señaló, las tra-

Limitaciones del método del circuito magnético 59

yectorias de los flujos de fuga se encuentran principalmente en regiones no magnéticas del espacio y las inductancias asociadas a estos flujos (inductancias de fuga) son elementos de circuitos lineales. 4. Saturación; todos los materiales y dispositivos de ingeniería exhiben cierto tipo de saturación, cuando deja de aumentar la salida al incremementarse la entrada, como sucede en la saturación de un amplificador electrónico. La saturación magnética, definida en la sección 2.2, es una parte de la característica no lineal descrita en el párrafo anterior, aunque generalmente se trata como característica separada de un circuito magnético. La saturación es de mucha utilidad en numerosos dispositivos electromagnéticos, tales como amplificadores magnéticos y reactores saturables. A los materiales magnéticos usados en este tipo de dispositivos magnéticos de conmutación yen otros, se les denomina materiales de ciclo cuadrado, pues sus ciclos de histéresis (figura 2.2) son aproximadamente cuadrados o rectángulares, con lados paralelos a los ejes B y H. La hipótesis de esta característica de ciclo permite un método de análisis'matemático de circuitos magnéticos directo. En casi toias las máquinas rotatorias es un aspecto más bien indeseable, que debe tenerse muy en cuenta en el diseño y operación de las máqui llas. En circuitos magnéticos complicados, la saturación se presenta generalmente en una parte del circuito, antes de tener lugar en otras. La operación de un circuito magnético dentro de la región saturarada de su curva B-H se traduce usualmente en baja eficiencia en energía y en calentamiento indebido. La frontera entre la parte no "saturada" y la "saturada" de la curva B-H se conoce como rodilla de esa curva y es la región donde la curva está más pronunciada hacia la horizontal. En todos los materiales, con excepción de los de ciclo cuadrado, la rodilla es un efecto gradual. 5. Flujos de fuga y de borde; ésta es una propiedad de todos los circuitos magnéticos. Es un tema que se trata mejor como parte de la solución generalizada de la distribución de campo magnético en el espacio, denominada a menudo problema de valores en la frontera. En muchos circuitos magnéticos de máquinas rotatorias, las fronteras entre regiones del espacio diferentes de materiales magnéticos (una frontera entre un material ferromagnético y el aire, generalmente) son a menudo superificies planas o cilíndricas, que se convierten en líneas rectas o círculos en sección transversal bidimensional. Las inductancias de fuga pueden calcularse en tales regiones, determinando la reluctancia o permeancia de la región mediante fórmulas integrales sencillas. Los coeficientes especiales o geométricos se conocen como coeficientes de permeancia. En la sección 2.10 se da un ejemplo de este método.

50 Circuitos magnéticos de CA. y C. C. estacionarios

2.7 EL CIRCUITO MAGNETICO IDEAL El circuito magnético ideal se puede definir mediante las hipótesis discutidas en la sección anterior; el circuito magnético ideal está compuesto por materiales magnéticos homogéneos, iso trópicos y lineales, y posee una permeabilidad infinita. Generalmente, se ignora el efecto de borde en el entre hierro de aire; un material magnético con permeabilidad infinita es análogo a un conductor eléctrico perfecto; esto es, un conductor con conductividad eléctrica infinita; las relaciones entre B y H son análogas a las que existen entre J y E en el conductor perfecto. Estas relaciones se establecen en casi todos los textos de teoría del campo y aquí sólo se sintetizarán. Obsérvese la figura 2.10, que muestra la frontera entre dos regiones de diferente permeabilidad magnética. La región 1 es el espacio libre con 111 = 110; la 2 es el material magnético con 112 -+ 00 • Las condiciones de frontera, en función de las componentes de campo H n y B n' normales a la frontera y las tangenciales a la misma H t y Bt se demuestra que son (presuponiendo que no hay densidad de corriente en la superficie de frontera):

B'n = B2n (2.23)

donde los subíndices 1 y 2 se refieren a las regiones respectivas. Dentro del material magnético donde la región 2, B 2/ H 2 / = -~O( fL2~OO)

(2.24)

fL2

muestra que la componente tangencial de la intensidad de campo en el material magnético es cero, a medida en que la permeabilidad se aproxima al infinito. Por consiguiente, partiendo de (2.23), la componente tangencial del campo magnético en la región 1, en la frontera, es también cero. Además, en la ecuación (2.15), puede verse que M,el potencial magnético, es nulo a lo largo de una trayectoria paralela al campo tangencial, dentro del material magnético. Hay dos conclusiones importantes de este análisis: 1. Las líneas de flujo o líneas de flujo magnético son perpendiculares a la superificie de un conductor magnético perfecto. 2. No hay diferencia de potencial o caída de reluctancia entre puntos o planos en diferentes posiciones, dentro de un conductor magnético perfecto.

Estas características del circuito magnético ideal se usan frecuentemente, en el análisis de circuitos magnéticos.

Ley de Faraday y voltaje inducido 61

FIGURA 2.10 Frontera entre materiales magnéticos

2.8 LEY DE FARADAY Y VOLTAJE INDUCIDO· Para relacionar una variación en el tiempo del flujo magnético, con una variación de campo eléctrico alrededor de una trayectoria cerrada, volvemos a la ley de Faraday, ecuación (2.2) de la sección 2.1:

~E.dl= -

f aa~

·ds

(2.2)

s

Con el objeto de determinar la dirección del vector de campo eléctrico E, se usan los dedos de la mano derecha para indicar la dirección positiva respecto a la trayectoria cerrada. El pulgar indicará la dirección de ds; una densidad de flujo B en la dirección de ds, aumentando con el tiempo, se traduce en una dirección para E, opuesta a la dirección positiva respecto a la trayectoria cerrada. El primer miembro de la ecuación (2.2) se denomina "fem" o voltaje inducido e. El derecho se definió en la ecuación (2.16) como el flujo magnético. Con estas definiciones, la forma escalar de la ley de Faraday es d

e=--

dI

(2.25)

Se define A, eslabonamientos de flujo, como el producto del número de vueltas N por el flujo que eslabona con N: (2.26)

Si la integral de línea en la ecuación (2.2) se evalúa alrededor de N trayectorias cerradas, que representan N vueltas en serie, la ecuación (2.25) se convierte en dA .

e=--

dI

(2.27)

62 Circuitos magnéticos de C.A. y C.C. estacionarios

La ley de Faraday es aplicable al caso en que la variación con el tiempo del voltaje provenga de un flujo variable en el tiempo, eslabonando a una bobina estacionaria, como en un transformador, una bobina o conductor, moviéndose físicamente a través de un flujo estático, en cualquier combinación de las dos situaciones. Para el caso de un conductor que se desplace en un campo magnético, el voltaje inducido se denomina a menudo "fem de movimiento" y se define como fem de movimiento ~(U X B)·dl

(2.28)

donde U es el vector velocidad del conductor. Una aplicación especial de esta relación es útil en el análisis de máquinas rotatorias. Si un conductor de longitud 1, se mueve perpendicularmente a la dirección de un campo magnético uniforme, con una velocidad constante la ecuación (2.28) se transforma en fem de movimiento = BlU

(2.29)

La dirección asociada a la fem de movimiento, se puede determinar a partir de los procesos de integración y reglas vectoriales expresadas en el segundo miembro de la ecuación (2.28), de manera semejante al proceso indicado por la ecuación (2.2), aunque cuando es aplicable la ecuación (2.29), es preferible seguir una regla mucho más fácil, conocida como regla de la mano derecha: colocar los dedos pulgar, ídice y medio de la mano derecha perpendiculares entre sí. Si el pulgar representa la dirección de U y el Índice la de B, el medio representará la dirección de la fem a lo largo de l.

2.9 RELACIONES ENERGETICAS EN UN CAMPO MAGNETICO

La energía potencial en un campo magnético se deteermina en el espacio mediante la integral de volumen

w= -1 f 2

vol

f. 2

B·Hdv= -1

/lB 2dv= -1 vol 2

1

2

B -dv vol JL

(2.30)

Ejemplo 2.4

Determinar la energía magnética potencial en el entrehierro de aire y en el material magnético del circuito magnético del ejemplo 2.2. De acuerdo a las hipótesis aplicadas en ese ejemplo, la distribución de campo era uniforme tanto en el entrehierro, como en el material magnético, facilitando el

Inductancia 63

uso de la ecuación (2.30). En el entrehierro, Bg = 0.001/(1.1 X 0.0016) 0.57 teslas. 2

w=l.(Bi)m puede sustituirse mediante la ecuación (2.17), por Mm' potencial magnético de la bobina m, dividido entre la reluctancia R del circuito magnético; el potencial magnético de la bobina m es N mIm . Realizando estas sustituciones en la ecuación (2.32) se obtiene L km =

kNkNm R =kNkNmP

(2.34)

donde P es la permeancia, recíproca a la reluctancia. En un circuito magnético simple, como el toroide, (figura 2.11), se puede sustituir la reluctancia dada por (2.18) en (2.34), dando (2.35)

Inductancia 65

donde 111' Al' Y lt son la permeabilidad, el área del corte transversal y la longitud media del toroide, respectivamente. La energía almacenada puede expresarse en función de la inductancia:

(2.36) Al sustituir L, dada por la ecuación (2.31) y Ni potencial magnético, dado por la ecuación (2.17), la (2.36) puede expresarse como

w= .lR2 2

(2.37)

Aquí cabe comparar estas expresiones de energía, con las expresadas en función de magnitudes de campo, dadas en la ecuación (2.30), de la sección anterior. Ambas formas son equivalentes. Ejemplo 2.5

Calcular la auto-inductancia de la bobina 1 del ejemplo 2.2 (figura 2.8). A fin de obtener un valor numérico para la inductancia, se debe especificar el número de vueltas de la bobina. Supóngase que son 10.

N,, (10)(0.001) L" = -/-,- = 5.04 =0.00198 henry Si se tomaran 100: L" =

(100)(0.001) 0.504 = 0.198 henry

N ótese que la inductancia varía con el cuadrado del número de vueltas. Ejemplo 2.6

Calcular la inductancia mutua entre las bobinas 1 y 2, del ejemplo 2.2., suponiendo 10 vueltas en la 1 y 20 en la 2. Dado que se ignoraron las fugas en ese ejemplo, k = 1.0; es decir, todo el flujo producido por la bobina 1 eslabona a la 2:

L 2r = L'2 =

(20)(0.001) 5.04 = 0.00398 henry

Ejemplo 2.7

Calcular la inductancia de la bobina 1 del toroide de la figura! 2.11, suponiendo que el material es una aleación de 48% de Ni y que la circunferencia

66 Circutos magnéticos de CA. y C. C. estacionarios

media del toroide es 0.1 m, con una área de la sección transversal igual a 0.0016 m 2 y con un factor de pila de 0.9. El número de vueltas es 10. De la curvaB-H para una aleación al 48% Ni, en la figura 2.4, se observa que la permeabilidad absoluta en amplitud es 0.115 en el intervalo lineal. Usando la ecuación (2.35) con k = /1.0, se obtiene

L(( =

(HY)(0.OOI6)(0.115) 0.1 =0.184 H

Como se ve, la inductancia de este toroide es mucho mayor que la del circuito de la figura 2.8, usado en el ejemplo 2.5, aunque las longitudes medias, áreas y vueltas de devanados sean las mismas. Esto se atribuye al efecto del entrehierro en el circuito de la figura 2.8 y también a la permeabilidad mayor del toroide.

Ejemplo 2.8 Determinar la inductancia de la ranura de armadura, cuya sección transversal se muestra en la figura 2.12, suponiendo que la parte baja de la ranura de altura y 2 está llena de conductores que llevan una densidad de corriente J ampere/m 2 , perpendicular al plano del papel. Supóngase también que el material magnético de acero que rodea la muesca tiene una permeabilidad magnética infinita. En este ejemplo se calcula una expresión para la "inductancia de ranura" que es una componente importante de la inductancia de fuga en muchos tipos de máquinas rotatorias, también se ilustra el uso de los coeficientes de permeancia y el concepto de eslabonamientos de flujo parciales. Se expone además el uso de la simetría, pues la figura 2.12 representa la sección transversal de una parte de una armadura de longitud la' en la dirección perpendicular al plano del papel. Es posible el análisis bidimensional, ya que sólo existe una componente de la densidad de corriente J. Suponer una permeabilidad finita del material magnético en los lados y base de la ranura significa que las líneas de flujo emergen perpendicularmente de los lados. Por consiguiente, las líneas de flujo pueden considerarse como componentes horizontales. Eslabonamiento de flujo parcial, es el término utilizado para describir las líneas de flujo que eslabonan sólo una porción de un conductor eléctrico o una parte de las vueltas de una bobina. En este ejemplo, una línea de flujo a través de la ranura a una altura cualquiera menor a Y2' eslabona únicamente con la corriente abajo de la línea de flujo. (La trayectoria de la línea de flujo se cierra a través del material magnético, como se muestra). Se debe haber observado que las supuestas líneas de flujo horizontal no son del todo correctas en una configuración práctica, pero sería imposible obtener una solución analítica sin esta conjetura.

Inductancia 67

Material

Conductor de la densidad de la corriente J

FIGURA 2.12 Sección transversal de una ranura

Analizar el flujo diferencial en la "franja" a una distancia y de la base de la ranura; la franja tiene una altura dy, una anchura la (hacia el papel) y una logitud t 2 • El potencial magnético que encierra esta franja es

La permeancia de la franja es

El flujo a través de la franja es, de acuerdo con la ecuación (2.17):

El flujo total a través del entrehierro es

El potencial magnético total de la ranura es

La permeancia de la porción inferior de la ranura es

y la de la parte superior

68 Circuitos magnéticos de C. A. y C. C. estacionarios

La inductancia de la ranura es

2.11 CIRCUITOS MAGNETICOS QUE CONTIENEN IMANES PERMANENTES

El segundo tipo de fuente de excitación que se usa comúmente para suministrar energía a circuitos magnéticos y a otros tipos de dispositivos mecánicos es el imán permanente. La naturaleza de estos imanes se describió brevemente en la sección 2.2. Existe desde luego, marcada diferencia entre una bobina de excitación eléctrica y una fuente de imán permanente, por lo que habrá ciertas diferencias en los métodos de análisis usados en los dos tipos de circuitos magnéticos. En realidad, estas diferencias son relativamente pequeñas y se relacionan con el mismo uso del imán permanente, más que con las otras partes del circuito magnético. Una bobina de excitación eléctrica ~nergizada por una fuente de voltaje o corriente continua sufre compautivamente poco la influencia del circuito al que excita, salvo durante e ondiciones transitorias, cuando se producen cambios en el circuito magnético o en el circuito eléctrico externo. En condiciones estacionarias, con una fuente de voltaje constante, la corriente en la bobina se determina solamente por la magnitud de la fuente de voltaje y la resistencia a la c.c. de la bobina. En un circuito excitado con un imán permanente, las condiciones de operación de éste las determina en gran medida el circuito magnético externo. Además, el punto de operación y la operación subsecuente del imán dependen de la forma en que éste se instala físicamente en el circuito y de si se magnetiza antes o después de instalarse. En muchos casos el imán debe pasar por una rutina estabilizadora antes de usarse; por supuesto, estos aspectos carecen de importancia con las fuentes de excitación eléctricas. Los detalles necesarios para encontrar la excitación requerida para establecer cierta densidad de flujo en un entrehierro de aire de dimensiones conocidas se indicaron el los ejemplos 2.1 y 2.2 para bobinas de excitación eléctrica. Por lo que respecta a una excitación por imán permanente, la meta es determinar las dimensiones del imán (longitud y sección transversal). El primer paso en este proceso es escoger un tipo específico de imán permanente, pues cada tipo de imán posee una característica única que condiciona parcialmente las dimensiones. del imán requerido. En un dise-

Circuitos magnéticos con imanes permanentes 69

ño práctico, esta elección se basa en factores de costo, disponibilidad, diseño mecánico (requisito de dureza y resistencia ), espacio disponible en el circuito magnético, así como en las especificaciones de operación magnética y eléctrica del circuito. La mayoría de los imanes permanentes no son maquinables y por lo general deben usarse en el circuito tal como los entrega el industrial. La tabla 2.3 resume algunas de las características pertinentes de los imanes permanentes comunes. La excitación por imán permanente se selecciona para una densidad de flujo especificada para el entrehierro, con la ayuda del segundo cuadrante de la curva B-H, llamada frecuentemente curva de desmagnetización para un tipo específico de imano Esta curva se introdujo en la sección 2.2 como figura 2.3. En la figura 2.13 se muestran las características B-H de algunos imanes permanentes de alnico y en la 2.14 las de algunos imanes de ferritas. También, en esta figuras se exponen las curvas de producto de energía (el producto de B en Gauss por H en Oersteds) y de razón de permeancia (relación de B entre H). El producto de energía es una medida de la energía magnética que el imán permanente es capaz de suministrar a un circuito externo, en función de su densidad de flujo e intensidad de campo. En general, un imán permanente se usa con el máximo de su eficiencia, cuando se opera en las condiciones de B y H que se traducen en un producto máximo de energía. Los coeficientes de permeancia son de utilidad en el diseño del circuito magnético externo. Se habrá observado que este parámetro en realidad es una permeabilidad relativa, como se definió antes, pues, 110 es igual a 1. O en el sistema CGS de unidades. Los símbolos Bd para la densidad de flujo magnético y Hd para la intensidad de campo seusanparadesignar las coordenadas de la curva dedesmagnetización. Una vez que se seleccionó el tipo de imán permanente, el diseño de las dimensiones del imán obedece el método general de la sección 2.4. De la ley de Ampére,

(2.38) donde

Ift == ~,

intensidad de campo magnético del imán, en Oe.

= longitud del imán, en cm

intensidad de campo en el entrehierro = densidad de flujo en el entrehierro (en unidades CGS) ~= longitudes del entrehierro, en cm ~nl::: caída de reluctancia en otras porciones ferromagnéticas del circuito, en Gilberts ~=

El área de la sección transversal del imán se calcula a partir del flujo requerido en el entrehierro, de la siguiente manera:

-.J

o

(j

::; ..., . ::::

ªo

Tabla 2.3 Características de los imanes permanentes

Tipo

3'"

Densidad de Fuerza coercitiva Prad. de energía máx. Permeabilidad de He (oersted) (gauss-oersted X 1

o' '"

~

~

i l~'

Razón de permeancia

8.5

9

9.5 10

11

12

13 14 15 16

6.0 ~ 5.5 ~ 5.0 ; :

14. 13. 12. 11. 10. 9. 8. ¡....-

~ V ,../ / ' / ' V V / / / V V VV / ' ,../ V .,,-V /" V / --- ~ ., -..... ." V V ¡...-.-- / ,../V ~ ~ / ~ ~ -..... es ~ /' ¡.......-- V .....-~ ~ .,/" __ ,~ 7.

4.0 ; :

~

~ ~

~x

_

1.5 _

~ 6. ~

-=_

0.5 _.

~ 8 g

0-

.t

1.0

75

150

14

¡.. 12

10

,/

4.5 .;:

2.0

40 50

/ 1/ 1 L L Vi / /y/ / ,...., / '/ / V VI /6J.. :r. I ¿ '/ V '¡; / V / ~ / L rr 1/ r /5 í/ V / V 1/ 1 / '/ r/ / V V '/ / / V V / / / J I / I !A ~ V V / /" / ' V V / / / I / )' / i/ I

6.5 '-....

2.5

30

14. 13. 12. 11. 10. 9. 8. 7. 6. 5. 4. 3. 2. 1.

7.0 ~

3.0

20 22 25

Producto de energía (Bd Hd X 106 )

"

7.5 ~

3.5

18

'>', ,,"'" ,,"'" " " " '\ "'\ \ \ \\ \ \ \ \ \\ \ \ \ \\ \\\\\\ \ \ ¡\ ¡\

8.0 .::- ""

_ -g

----- -- ....---::..- -----

f-- r-

5.

1-

4. 3. r2 1.

~V

f--

~

1--1--

/

1600

1200

1400

~

~

1000

I

!L V

.-- V

1('

V Y /j. VJ

rTL J

I/i / 600

6

'/

~v

o

:>

1

V

0.4

0.2

O

FIGURA 2.14 Curvas de desmagnetización y de productos de energía para imanes de cerámica Indox. Clave: 1, Indox 1; 2, Indox 11; 3, Indox V; 4, Indox VI-A

o

-g

72 Circuitos magnéticos de C.A. y

c.c. estacionarios (2.39)

donde Bd = densidad de flujo en el imán, en Gauss A m = área de la sección transversal del imán, en cm 2 Bg == densidad de flujo en el imán, en Guass Ag = área de la sección transversal del entrehierro, en cm 2 • K 1 = factor de fuga

El factor de fuga K 1 es la razón del flujo que emerge del imán entre flujo en el entrehierro. La diferencia entre estos dos flujos es el flujo de fuga en las regiones del espacio entre el imán y el entre hierro de aire. El factor de fuga puede determinarse por los métodos descritos en secciones previas de este capítulo o mediante otros métodos estándares más exactos. A través del tiempo se han establecido cierto número de fórmulas para las configuraciones comunes utilizadas en circuitos magnéticos deimán permanente y pueden encontrarse en las referencias bibliográficas 1,2 Y 3. Dos de estas fórmulas se ilustrarán en los ejemplos que siguen. Ejemplo 2.9 Determinar la longitud y área de la sección transversal de los imanes de la figura 2.15 para producir una densidad de flujo de 2500 G en el entrehierro. El imán permanente que va a usarse es alnico V; las dimensiones en la figura 2.15 son: 19 = 0.4 cm, W = 6.0 cm y el área del entrehierro es = 4.0 cm 2 (2 cm X 2 cm). Se supone que la reluctancia en las porciones de hierro blando del circuito es insignficante, dando una caída de reluctancia V m i igual a cero. Se estima que el factor de fuga es de 4.0 y que el imán se operará en su condición de producto de energía máximo (rodillo de la curva de desmagnetización en la figura 2.13). De la ecuación (2.39): Am =

BgAgK¡ (2500)(4.0)(4.0) 2 Bd = 10,500 = 3.8 cm

De la ecuación (2.38), observando que Hg

1m =

= Bg

en unidades CGS,

(2500)(0.4) . 450 =2.22 cm= H (en la fIgura 2.15)

En la figura 2.15, la dimensión lm determina lasdimensionesH y G. Ahora, se debe comprobar la hipótesis, sobre el factor de fuga. De la ref. 2, se to-

Circuitos magnéticos con imanes permanentes 73

Imán

~I~~-------w--------~>~I FIGURA 2.15 Circuito magnético con imán permanente

ma un factor de fuga para esta configuración, que se basa en mediciones experimentales y cálculos semejantes a los descritos previamente en este capítulo, con un valor

(2.40)

donde CH Cw Ce

=

perímetro de la sección transversal del circuito de long. H

= perímetro de la sección transversal del circuito de long. W = perímetro de la sección transversal del circuito de long. G

El factor 0.67 en el tercer término dentro del paréntesis rectangular en la ecuación (2.40) procede de la característica de los imanes permanentes de tener una "zona neutral" que no contribuye a la figura. A partir de los calculos de longitud y área anteriores, es posible calcular los parámetros de longitud en la ecuación (2.40) y estimar los perímetros de las diversas secciones transversales: H = 2.22 cm, G = 0.91 cm, W = 6.0 cm, Ce = 8 c, Cw = 8.0 cm y CH = 7.8 cm. Sustituyendo estos valores en la ecuación (2.40) se obtiene K] =4.062

Este valor puede introducirse en la ecuación (2.39), originando un cambio ligero en el área A In del imán. A la vez este cálculo puede requerir algunos

74 Circuitos magnéticos de C.A. y C.C. estacionarios

cambios en otras dimensiones usadas en la ecuación (2.40), dando un nuevo valor para el factor de fuga. Algunas iteraciones de estas fórmulas, por lo general son necesarias para obtener un conjunto uniforme de dimensiones para el circuito magnético total. El valor elevado del factor de fuga obtenido para la configuración de la figura 2.15 indica que éste no es un circuito magnético muy eficiente. Dicho de otra manera, el imán permanente se encuentra en una posición errónea dentro del circuito magnético. La fuga alta en esta configuración puede fácilmente explicarse por medio de la ley de Ampére y la teoría elemental de los circuitos magnéticos presentada con anterioridad en este capítulo. Un uso mucho más eficiente del imán permanente en esta configuración se obtiene colocándolo adyacente al entrehierro, como se muestra en la figura 2.16. De la Ref. 2, el factor de fuga para la figura 2.16 es

(2.41)

Con las mismas dimensiones para todas las secciones del circuito de la figura 2.16 como se usaron para la ecuación 2.15 (aunque esto pudiera resultar en un imán permanente de dimensiones excesivas), al sustituirlas en la ecuación 2.41 se tiene

K¡ = 1.624 que es menor a la mitad del factor de fuga encontrado para la configuración de la figura 2.15 Es interesante ver el volumen de material de imán permanente necesario para establecer un flujo dado en el entrehierro de aire. Al resolver para Am en la ecuación (2.39) y para 1m en (2.38) (sin considerar a V mi) Y observando que en el sistema CGS Hg = Bg, se obtiene

(2.42)

Se ve que el volumen de imán es una función del cuadrado de la densidad de flujo en el entrehierro. La importancia del factor de fuga para reducir al mínimo el tamaño del imán es obvia observando esta ecuación. El denominador de la ecuación (2.42) es el producto de energía, que es función del material de imán permanente y del punto de operación en la curva de desmagnetización del imán. El parámetro razón de permeancia, mostrado en las figuras 2.13 y 2.14, es la razón de la permeancia equivalente del circuito externo,AgK¡ /lg en-

Circuitos magnéticos con imanes permanentes 75

FIGURA 2.16 Uso eficiente de imanes permanentes en un circuito magnético.

tre la permeancia del espacio ocupado por el imán permanente A m 11m en el sistema CGS de unidades. Esto puede observarse resolviendo para Bd en la ecuación (2.39) y para Hd en la (2.38), (sin tomar en cuenta a V mi) Y formando la razón

(2.43)

La ecuación (2.43) es tan simple en apariencia que puede engañar, ya que el trabajo para obtener una expresión analítica para K 1 es muy difícil, como se vio. Además, la caída de reluctancia V m i en las partes de hierro, blando del circuito magnético debe incluirse de alguna manera en la ecuación (2.43). Esta es una tarea aún más difícil, pues la caída de reluctancia es función del punto de operación del imán permanente Bd Hd Y de los efectos del flujo de fuga en el hierro. La caída de reluctancia se introduce por lo general mediante un factor similar al factor de fuga y se basa en mediciones de configuraciones de circuitos prácticos. Las diversas expresiones en la ecuación (2.43) tienen valor al observar las relaciones generales entre parámetros magnéticos al variar la permeancia del circuito externo. Esto conduce al segundo tipo de imán permanente. Los circuitos con entrehierro variable se describirán brevemente con la ayuda de la figura 2.17. Sin olvidar la ecuación (2.43), obsérvese la variación de B y H de un imán permanente, al modificarse la permeancia del circuito externo. La figura 2.17 muestra una característica B-H típica de segundo cuadrante para un imán permanente. En teoría, es posible tener una permeancia infinita en el circuito magnético externo que corres-

76 Circuitos magnéticos de C.A. y C. C. estacionarios

H (Oe)

FIGURA 2.17 Característica B-H de segundo cuadrante para un imán permanente.

pondería a ~ = 90° en la ecuación (2.43), con un punto de operación de imán en Bd = Br Y Hd = O de la figura 2.17. Este sería aproximadamente el comportamiento de un imán permanente con circuito externo, sin entrehierro y con un miembro de hierro blando de alta permeabilidad, llamado a menudo "conservador". En la práctica, sin embargo, siempre existe un pequeño entrehierro equivalente y una caída de reluctancia reducida en el conservador y el punto de operación se encuentra a la izquierda de Br Y ~ es menor de 90° . Para un entrehierrofinito, el punto de operación se encuentra en algún punto A de la curva B-H y el imán permanente creará la intensidad de campo magnético HA para compensar la caída de reluctancia en el entre-

Circuitos magnéticos con imanes permanentes 77

hierro yen otras porciones del circuito magnético externo. Si se incrementa el entrehierro, Pge se reduce y el imán debe generar una intensidad de campo magnético mayor; Hd. En la ecuación (2.43) se ve que a decrece y que el punto de operación en la figurar 2.17 se desplazará más a la izquierda hasta el punto B en a B. Si el entre hierro regresa en seguida a su valor original, el punto de operación no regresa a A, sino más bien a C. Si el entrehierro varía sucesivamente entre los dos valores, el punto de operación trazará un ciclo de histéresis menor entre B y C, como se muestra en la figura 2.17. La pendiente de este ciclo se conoce como permeabilidad de rebobinado; dado que se trata de una pendiente en el plano B-H, algunas veces se le llama permeabilidad incremental, como se definió en la ecuación (2.11). Esta premeabilidad es un parámetro importante de los imanes permanentes en aplicaciones de entrehierros variables. En la tabla 2.3 se dan valores de este parámetro para los imanes permanentes expuestos. Bibliografía 1. H. C Roters, Electromagnelic Deuices, John Wiley & Sons, Inc., Nueva York, 1941. 2. "Design and Applications of Permanent Magnets" Indiana General Corporal Ion Manual No. 7, Valparaiso, Indiana, 1968. 3. "Permanet Magnet Design", Thomas and Skinner, Inc, Bulletin M303 Indianáp >lis, Indiana 196'1. 4. L. V. Bewley, Two-Dimensional Fields in Electrical Engineering, Macmillan Publishing Co., Inc., Nueva York, 1948. 5. P. Silvester, Modern Electromagnetic Fields Prendice -Hall, Inc_, Englewood Cliffs, N. J., 1968.

Problemas 2.1 De las características materiales magnéticas mostradas en la figura 2.4, determinar la permeabilidad en amplitud relativa, en una densidad de flujo de 1.0 T para los siguientes materiales: a) Deltamax, b) M-19 Y e) AISI 1020. 2.2 En la sección 2.3 se estableció que el área comprendida por el ciclo de histéresis en c.c. de un material magnético, es igual a la pérdida por histéresis de ese ma· terial. Con referencia a la figura 2.2, encontrar las unidades SI de esta pérdida a partir de las coordenadas del ciclo de histéresis. 2.3 En la figura 2.4, determinar la permeabilidad diferencial relativa para el material 48NI, en una densidad de flujo de 0.8 T. 2.4 El acero al silicio al 3% designado como M-19 en la figura 2.4 se usa mucho en transformadores de potencia pequeños, en motores de inducción y en otros dispositivos electromagnéticos utilizados en circuitos electrónicos. En este capítulo se estableció que hay una parte de la característica B-H, en la cual la relación es aproximadamente lineal. El'to no se aprecia necesariamente en la figura 2.4, pues se trazaron las curvas en papel semilogarítimico con el objeto de describir la característica en un intervalo más amplio. Trazar de nuevo la curva M-19 en papel de

78 Circuitos magnéticos de C.A. y C,c. estacionarios

coordenadas rectangulares y determinar el intervalo de B, dentro del cual la característica pueda considerarse lineal. ¿Cuál es la permeabilidad relativa en este intervalo? 2.5 Derivar una ecuación para describir la línea recta o porción lineal de la curva B-H para M-19, obtenida en el problema 2.4. 2.6 Si fuera necesario usar la característica. B-H de M-19, en un programa de diseño de computadora digital para un transformador en electrónica ¿cuál. sería el modelo de esta característica, incluyendo saturación y características iniciales? 2.7 Discutir el significado físico de la primera ecuación de Maxwell (2.1), en base a la naturaleza de las líneas de flujo magnético (líneas de corriente magnética). 2.8 Un entrehierro tiene un área de una pulgada cuadrada y una longitud de un mm. ¿Cuál es la reluctancia y la permeancia de este entrehierro en unidades SI? 2.9 En el entrehierro del problema anterior existe un flujo de 0.02 Wb. Suponiendo que no hay efectos de borde, ¿cuál es el potencial magnético (FMM) a través del entrehierro? 2.10 Tómese una región del espacio descrita por coordenadas cilíndricas convencionales r, ,/

1\

l

,

I \

I

\

f

2/1 \

s,

\

f

+

.......... N2

\~+-+-

"-"-----~

,------------_/

V2

______________-'

I I

FIGURA 3.5. Modelo de transformador en el que se muestran voltajes, corrientes y flujos instantáneos.

En esta sección se considerará un transformador excitado con una fuente de voltaje sinusoidal en una sola frecuencia, denotada con v I en la figura 3.5. Se analizarán en primer término los voltajes, corrientes y flujos magnéticos de régimen estacionario. En secciones posteriores de este capitulo se estudiarán las características transitorias del transformador. Se iniciará este estudio con las relaciones de campo electromagnético en el transformador, como una ayuda para comprender los fenómenos físicos que se traducen en características externas del transformador. No es menestar utilizar la teoría del campo magnético para describir las características externas del transformador, que pueden analizarse totalmente mediante la teoría de los circuitos eléctricos. Los métodos de esta teoría resultan adecuados para una gran cantidad de problemas de ingeniería eléctrica, tales como cuando el transformador se reperesenta por un modelo matemático o como un caudripolo o "caja negra". Este método de circuitos se discutirá posteriormente con más detenimiento en este capítulo, de acuerdo con los desarrollos del circuito equivalente del transformador. 3.2.1 Características de excitación En la figura 3.5, supóngase que el conmutador S está abierto y que i2 = O. Esta condición se denomina condición sin carga y facilitará el establecimiento de las relaciones sin carga o de voltaje y corriente de excitación. Se supondrá que el voltaje aplicado es

(3.2) donde VI m = valor máximo de voltaje aplicado w = 2rrt = frecuencia angular del voltaje aplicado t = frecuencia en Hz.

Teoría del transformador 93

Suponer, de momento, que la resistencia del devanado NI, es nula. Entonces

v,+e,=O ó

(3.3) donde el

= voltaje inducido electromagnéticamente en el devanado NI

De (2.27) del capítulo 2, este voltaje puede expresarse como

(3.4) donde '(B)=L(B)i donde i es la variable independiente. Por consiguiente, de (4.11),

154 Sistemas electromecánicos

la que, expresada en términos ·de inductancia y corriente, se convierte en

Para variaciones dadas de corriente e inductancia,

Si se permite que el rotor gire con una velocidad angular w m tal que en cualquier instante O=Wmt -

o

.(o es la posición del rotor en t = O, cuando la corriente i es también cero),

la expresión para el par instantáneo queda entonces en términos de Ü)m como Te = -

W

y

~ I~L' {sen2(wmt- o) - ~ [sen2(wmt +wt- o)

+sen2(wm t -wt - o) ] }

Para obtener la forma final anterior, se usaron las siguientes identidades trigonométricas: sen2A = -!(1-cos2A) y

sen ecos D

1

= 2' sen (e + D) +

2'1 sen (e -

D)

De la expresión anterior se concluye que el par promediado en el tiempo es cero, ya que el valor de cada término integrado en un período es cero. El único caso en que el par promedio no es cero es cuando w = W m • Para esta frecuencia particular, la magnitud del par promedio es L' sen_o ')< T prom -- 4.1]2 m

o, de la figuro:) 4.6b,

i

Tprom = I~ (L d

-

L q )sen2ú

Así, por ejemplo, en 1m = 4 A, Ld = 0.2 H y Lq máximo es 0.2 N-m (Newton-metro).

= 0.1 H y el par promedio

La ecuación de fuerza 155

Pueden sarse varias conclusiones del análisis anterior. La máquina desarrolla un par promedio sólo en una velocidad particular, correspondiente a la frecuencia w = w m , que se conoce como velocidad síncrona. La máquina de reluctancia es por lo tanto una máquina síncrona. El par desarrollado por la máquina se denomina par de reluctancia, que será nulo si Ld = Lq . El par varía sinusoidalmente con el ángulo o, llamado ángulo de par. Las inductancias Ld y Lq son los valores máximo y mínimo de la inductancia y se les conoce como inductancia de eje directo e inductancia de eje en cuadratura, respectivamente. El máximo par tiene lugar en el punto en que = 45°, llamado par de tirón exterior. Cualquier carga que requiera un par mayor que el máximo, se traduce en una operación inestable de la máquina.

o

Ejemplo 4.3 En este ejemplo se estudia el efecto de incluir en el análisis la resistencia de una bobina excitada con voltaje, como la de la figura 4.5. Sea r la resistencia de la bobina de N vueltas. Sin tomar en cuenta la saturación, determine los valores instantáneo y promedio de la fuerza eléctrica si el vol taje de terminal es v = Vm sen wt y la reluctancia del circuito eléctrico corrpleto puede expresarse como R = a + bx, donde a y b son constantes. La energía almacenada en el campo magnético se expresa así:

donde tiene

N1> = A es el eslabonamiento de flujo de la bobina. Por lo tanto, se

ya que R = a + bx. El problema se reduce ahora a relacionar 1> y v como sigue:

.

dcp

v=n+Ndi'

where cp=

Así, bajo condiciones de régimen estacionario,

donde

Ni R

156 Sistemas electromecánicos

y

La fuerza instantánea es, por consiguiente,

El valor promedio en el tiempo se determina de

Obsérvese que el valor promedio de sen 2 (o cos 2 ) es 1/2.

4.4 VARIACIONES DE CORRIENTE Y FLUJO En un sistema excitado con voltaje es necesario investigar la variación de la corriente de entrada en función del tiempo. La discusión siguiente es sólo cualitativa. Más tarde se verá un ejemplo numérico. Consideremos el sistema mostrado en la figura 4.7. Cuando no hay voltaje aplicado, el hierro móvil (armadura) se encuentra, digamos, a una distancia X o del núcleo. La inductancia correspondiente es Lo -valor mÍnimo de la inductancia- y la constante de tiempo 70 es Lo/r. Si se mantiene el hierro en la posición original y se aplica un voltaje escalón, el circuito se comporta como un circuito rL, con constante de tiempo 70 • La corriente final es V/r, donde Ves el voltaje aplicado. No obstante, si se le permite

¿(x)

FIGURA 4.7 Modelo de un relevador electromagnético.

Dinámica de los sistemas electromecánicos 157 Corriente final ~

r

2l e

"Eo U

t Principia el movimiento mecánico

FIGURA 4.8 i(t) para un voltaje de entrada escalón.

al hierro desplazarse y su posición final es xf, la inductancia del circuito aumenta a Lf y la constante de tiempo correspondiente es Tf = Lf/r. Es evidente que Tf > To . Para las posiciones inicial y final del hierro móvil, se muestran las corrientes en la figura 4.8 por las curvas a) y b), respectivamente. Sin embargo, la transición de a) a b) no es continua, porque tan pronto como el hierro principia a moverse, la constante de tiempo mecánica T m entra en juego. Debe reconocerse que T m > T f > T o. Esto explica la naturaleza de la variación de corriente cuando al hierro móvil se le permite moverse en un voltaje constante. En el caso de excitación con corriente constante, son de interés las variaciones de flujo. Para la posición inicial X o la reluctancia es máxima y el flujo correspondiente 1>0 es un mínimo. El flujo alcanza su valor máximo 1>f cuando se completa el movimiento. La variación del flujo la gobiernan tanto la constante de tiempo mecánica como la eléctrica, como en el caso anterior. Es interesante obtener la variación de flujo en función del tiempo. (Ver Probo 4.12).

4.5 DINAMICA DE LOS SISTEMAS ELECTROMECANICOS En las discusiones precedentes se supuso un movimiento mecáncio y eléctrico durante el proceso de conversión de energía electromecánica. En esta' sección se estudiará la dinámica electromecánica, derivando las ecuaciones pertinentes del movimiento y resolviéndolas en seguida para varias condiciones de operación. Se obtendrá así el comportamiento dinámico de un sistema dado. También se considerarán algunos ejemplos ilustrativos. Al tratar de obtener la información cuantitativa necesaria acerca de un dispositivo de conversión específico, por lo general surgen varias dificultades. En primer lugar, los parámetros involucrados en las ecuaciones de movimiento son difíciles de evaluar. Tienen que hacerse aproximacio-

158 Sistemas electromecánicos

nes liberales para obtener modelos susceptibles de análisis. Para citar un ejemplo específico, al calcular las inductancias de varios devanados de un dispositivo electromagnético, se menosprecian con frecuencia los efectos de saturación como una primera aproximación. Del mismo modo, en conversores rotatorios de energía no es conveniente tomar en cuenta ciertas consideraciones prácticas como armónicas, fugas y efectos de ranuras y dientes, a pesar de que cada uno de estos factores contribuye a la operación del dispositivo. En estos casos se reemplaza el dispositivo por un modelo idealizado. Se introducen supuestos para simplificar y cuando es necesario obtener cierto grado de exactitud, se incluyen los efectos de segundo orden en la solución, mediante técnicas especiales, como métodos gráficos y numéricos. Una vez determinados los parámetros en las ecuaciones de movimiento -un paso de suma importancia para estudiar los dispositivos de conversión de energía-, la formulación de las ecuaciones mismas (que se estudiará después) es un asunto de rutina. El segundo paso difícil en el estudio de un dispositivo de conversión de energía, es resolver las ecuaciones de movimiento obtenidas. Estas ecuaciones casi siempre no son lineales con coeficientes variables en el tiempo. No existen métodos generales para resolver este tipo de ecuaciones y es necesario hacer aproximaciones para obtener los resultados finales. Por ejemplo, en el caso de transductores, se supone un movimiento incremental (a menudo no demasiado fuera de la realidad) y se linealizan las ecuaciones no lineales resultantes, alrededor de un punto de operación en reposo. Una vez linealizadas, las ecuaciones se resuelven por un método usual. Para conversores de energía rotatorios, como los motores y generadores, el procedimiento para formular las ecuaciones de movimiento es semejante al del transductor. La solución de estas ecuaciones se facilita mediante transformaciones lineales o por alguna otra técnica, como métodos numéricos, o por el uso de computadoras analógicas o digitales, que también pueden utilizarse para resolver las ecuaciones de movimiento sin linealizarlas. El análisis y estudio de los conversores de energía electromagnéticos incluye, por lo tanto, los siguientes aspectos:

1. Consideraciones topológicas y descripciones físicas: proporcionan las ubicaciones de las terminales de entrada y salida; identifican los elementos móviles y fijos y especifican la estructura del circuito magnético, de datos de devanados y de otras dimensiones físicas. 2. Elección de un modelo y de supuestos para simplificar: Por lo general dependen del problema que se trate y del grado de exactitud deseado en la solución. Por ejemplo, en un dispositivo magnético, en una primera aproximación no se toman en cuenta los

Dinámica de los sistemas electromecánicos 159

efectos de borde, la saturación y la histéresis. Se supone que la permeabilidad del material magnético es infinitamente mayor que la del espacio libre y, en consecuencia, se supone que la energía magnética se almacena sólo en el entrehierro. 3. Determinación de los parámetros del sistema: aquí se incluye la evaluación de las resistencias, inductancias y capacitancias de la parte eléctrica del sistema, así como la determinación de la masa (o momento de inercia), rigidez y coeficiente de rozamiento para la parte mecánica. Los parámetros se obtienen casi siempre de la descripción física del sistema y de las suposiciones para simplificar la elección del modelo. En la mayoría de los casos de importancia práctica, la determinación de los parámetros consiste en el cálculo de varias inductancias, un paso difícil pero importante, del que depende el rendimiento de un dispositivo electromagnético. 4. Formulación de las ecuaciones de movimiento eléctrico y mecánico: estas son, respectivamente, las ecuaciones volt-ampere y las de equilibrio de fuerzas (o de equilibrio de pares). Estas ecuaciones se pueden derivar de uno de los métodos que se estudiarán después. 5. Solución de las ecuaciones de movimiento: este paso se efectúa una vez que se formulan las ecuaciones de movimiento. Casi siempre, las ecuaciones diferenciales resultantes son no lineales. En casos sencillos, como en el de transductores para aplicaciones de señales débiles, primero se linealizan las ecuaciones de movimiento y después se resuelven como si fueran ecuaciones diferenciales lineales con coeficientes constantes. En casos complicados de máquinas eléctricas rotatorias, son necesarios ciertos tipos de transformaciones lineales. No se dispone de un método general que abarque todos los casos. Si se dispone de computadoras digitales, es muy fácil obtener soluciones numéricas, aun sin linealizar las ecuaciones de movimiento. 4.5.1 Formulación de las ecuaciones de movimiento

Las características eléctricas y mecánicas de un sistema electromagnético las proporcionan, respectivamente, las ecuaciones de movimiento eléctrico y mecánico, que son las de equilibrio de voltajes (o corrientes) y de equilibrio de fuerzas, (o pares) que pueden obtenerse igualando las "fuerzas aplicadas" con las de "restauración". En la ecuación eléctrica, las fuerzas eléctricas de origen mecánico se obtienen de la ley de Farday. Las fuerzas mecánicas de origen eléctrico se determinan mediante la ecuación de fuerza establecida antes. Al formular la ecuación mecánica, se considera la fuerza mecánica que resulta del efecto de campo magnético

160 Sistemas electromecánicos

lE

~_ _

1

1

-

I I+----~---- x

I

Masa M

1

----

0

_j

-------- - ---- -

~

~ ~b

Coeficiente de rozam iento

a Sección transversal

FIGURA 4.9 Un sistema electromecánico.

como una fuerza aplicada exteriormente. Se denominará esta fuerza como una fuerza eléctrica y se designará con Fe' Con el ejemplo que sigue se ilustrará la formulación de las ecuaciones de movimiento. Un ejemplo de relevador electromagnético es el modelo de un sistema de parámetros concentrados, que se muestra en la figura 4.9. No hay fuerza mecánica aplicada exteriormente. Se desea, en primer lugar, formular las ecuaciones dinámicas de movimiento de acuerdo con los pasos siguientes: 1. Suposiciones. No se tomará en cuenta la saturación del circuito magnético, el cual se supone es de permeabilidad infinita, y también se ignorarán los flujos de fuga y de efectos de borde. Además, se supone que la fuerza de rozamiento es linealmente proporcional a la velocidad, y la del resorte también linealmente proporcional a la elongación. 2. Parámetros. Los parámetros mecánicos son: masa M, coeficiente de rozamiento b y la rigidez del resorte k. Los parámetros del circuito eléctrico son: resistencia r, e inductancia L, que pueden expresarse en términos de las dimensiones mostradas en la figura 4.9 como sigue: /LoaN

2

A

L(x)=--=-ll-x C+x

(4.19)

donde A yC son constantes. (Observar que A = -l1 o aN 2 y e = -tI)' 3. Ecuaciones de movimiento. Ahora se pueden identificar las diferentes fuerzas que actúan sobre el sistema. a) Eléctricas:

ri+

:r (Li)=v

(4.20)

Dinámica de los sistemas electromecánicos 161

donde los términos del primer miembro denotan a las fuerzas de restauración o caídas de voltaje. b) Mecánicas:

Mx+bx+k(x-l )=F = !"i2 oL o e 2 OX

(4.21)

donde el primer miembro es la suma de las fuerzas de restauración, y donde Fe, la fuerza eléctrica considerada como fuerza externa, se obtiene de la ecuación (4.11) o la (4.18). 4.5.2 Una reconsideración de (4.20) y (4.21)

Aquí se reconsideran las ecuaciones de movimiento, en vista de que son importantes para determinar el comportamiento de un sistema. En la ecuación eléctrica, el término que resulta del movimiento mecánico es de particular importancia y en la ecuación mecánica reviste especial interés el término que se atribuye al "movimiento eléctrico". Así, en la ecuación (4.20), se tiene d(li)jdt, que también puede escribirse como -d

dt

(L') ¡ -_ L -di dt

+¡ .dL . dL - -_ L -di + ¡ - X. dt

dt

dx

(4.22)

x

donde = dx/dt = velocidad mecánica, que es un cambio de la inductancia con respecto al tiempo. Obsérvese que el segundo término en (4.22) proviene del movimiento mecánico y se denomina voltaje de movimiento. Además, la presencia de los términos (4.22) en (4.20) la hace una ecuación diferencial no lineal. Más tarde se insistirá en este punto. Consideremos ahora el segundo miembro de (4.21), que junto con (4.19) da

li2_oL_ = lp_O_(_A_) = 2

OX

2

ox

e+ x

_

A _...:.i_2_ 2 (e + X)2

(4.23)

que es también un término no lineal que hace a (4.21) una ecuación diferencial no lineal. Nótese que A = -Ji Q aN 2 Y e = --/1' lo que implica que la fuerza eléctrica sea po~tiva y actúe en la dirección positiva de las x. 4.5.3 Soluciones anal íticas de las ecuaciones de movimiento

Por conveniencia, se reescribirán las ecuaciones de movimiento (4.20) Y (4.21) como sigue: di .dL. . L -+¡-x+n=v dt dx

(4.24)

162 Sistemas electromecánicos

(4.25)

Como se mencionó en la última sección, estas ecuaciones son no lineales y no es posible obtener soluciones analíticas explícitas. No obstante, para señales débiles y movimiento incremental, puede obtenerse información útil sobre el sistema resolviendo las correspondientes ecuaciones de movimiento diferenciales linealizadas. Además de la limitación a señales débiles, la linealización de las ecuaciones para un sistema físico dado, requiere que exista un punto de equilibrio estable. Las señales pequeñas (o movimiento) son los recorridos alrededor de este punto de equilibrio. Sea que (Vo , lo, X o ) denoten al punto de equilibrio estable en estado estacionario, tales que

v(t)= Vo+VI(t) i(t)=lo+il(t)

(4.26)

x(t)=Xo + xl(t) donde (v, i, x) son las variables originales y (v 1, Xl, i 1) pequeñas perturbaciones alrededor de (Vo , lo, X o ). La pequeñez se mide por hecho de que términos de tipo producto como ¡2, i 1 X 1 etc., son despreciables. Sin perder de vista estas limitaciones, se examinarán los términos no lineales en (4.24) Y (4.25). Por ejemplo, sustituyendo x = Xo + Xl en (4.19), se obtiene 2

2

/LoaN /LoaN 1 ) ( L= ll-X = II-Xo-x l =Lo 1-x l /(lI-Xo) (4.27)

X )-1

=Lo 1 ___1_ ll-Xo

(

donde Lo = [Jl o aN 2 /(11 - X o )]. Para valores pequeños de de desarrollarse en serie binómica como

Xl'

(4.27) pue-

1

(4.28)

I I )3 XI (X L=Lo 1+--+ - - )2 + (-X+ ... [ 11 - X o tI - Xo 11 - Xo Además, de (4.28) se obtiene

I )2 + ... aL= Lo- [ 1+--+3 2x I (X -aX I 11 - Xo 11 - X o 11 - X o

1

(4.29)

Dinámica de los sistemas electromecánicos 163 Las formas linealizadas de (4.28) Y (4.29) serían entonces

(4.30) y

(4.31)

donde se supone que XI / (1 1 - X o ) lo

~

Q)

:J lL

0.4

0.3 Fuerza elástica

0.2

0.1

2.1

2.2

2.3

2.4

2.5

2.6

2.7

Xo(cm) -

FIGURA 4.10 Fuerza contra distancia. I~ = 2.45 A;I~'= 20 A; 11 = 3 cm, 19 = 2 cm, N = 200 vueltas; a = 1 cm 2 ; 110 = 41T X 10- 7 H/m y k = 1 N/cm.

166 Sistemas electromecánicos

sorte y la eléctrica, para dos corrientes diferentes. Existe un punto de equilibrio si la línea que representa a la fuerza del resorte corta a la curva de la fuerza eléctrica. Así, para una corriente mayor que lo (2.5A) no hay punto de equilibrio y la fuerza eléctrica es siempre mayor que la del resorte, llevando a la masa de hierro a la posición extrema (Xo = 11 ). Para la corriente l~ en el punto 3 (Xo = 2.3 cm) la fuerza del resorte es igual a la eléctrica. Sin embargo, una ligera perturbación alrededor de este punto haría que la fuerza eléctrica excediera a la de restauración del resorte. En consecuencia, para I~ = 2.45 A, Xo = 2.3 no es un punto de operación estable. En seguida, si la corriente se reduce a li;' (= 2.0 A), obtenemos los puntos 1 y 2 como puntos de equilibrio. En el punto 2, si la masa se desplaza a la izquierda (Xo incrementada), la fuerza eléctrica se hace siempre mayor que la del resorte, originándose un movimiento continuo hacia la izquierda hasta que X o = 11 • Por otra parte, si la masa se moviera a la derecha, (Xo decrementada) la fuerza del resorte sería siempre mayor que la eléctrica y el movimiento continuaría hasta el punto 1 (Xo = 2.13 cm), que es el punto de equilibrio estable. Una perturbación a la izquierda o a la derecha del punto 1 haría que la masa regresara a 1. Por consiguiente, la operación estable en reposo para el sistema es (Va = 6 V; lo = 2 A; Xo = 2.13 cm). 4.5.5 Soluciones numéricas

El uso de variables de estado en la teoría de sistemas lineales es una práctica común y una formulación de ecuaciones en términos de estas variables facilita el estudio de los sistemas dinámicos. Los dispositivos electromecánicos, tales como transductores y máquinas rotatorias, se consideran como sistemas dinámicos. Con la ayuda de rutinas normales de computadora, puede obtenerse la solución de la ecuación de estado y estudiarse las características dinámicas del sistema. N o es más difícil programar la ecuación no lineal que el sistema lineal, por lo que se evita el paso de linealización. El análisis de un sistema no lineal es en realidad más fácil que el de la aproximación linealizada del sistema. Las ecuaciones que describen el comportamiento de un sistema electromecánico puede reducirse a la formulación de estado

y=f(y, v,t) donde y es el vector de estado y v es el vector de entrada. Si se supone un movimiento pequeño alrededor de algún punto estable, la ecuación de estado puede linealizarse como y=Ay+Bv

Dinámica de los sistemas electromecánicos 167

donde las matrices A y B se forman con parámetros del sistema. Como muestra el ejemplo, si se busca una solución por computadora, es más fácil operar con las ecuaciones no lineales. Para el relevador electromagnético se determi.narán la corriente, posición de armadura y la velocidad, en función del tiempo para una entrada de voltaje escalón. Sean y 1 = i, Y2 = X, Y3 = X las variables de estado en las ecuaciones de movimiento (4.20) y (4.21), con una L dada. Las ecuaciones de estado se convierten en

)

(4.41)

Las ecuaciones linealizadas (4.36) y (4.40) se expresan ahora como

(4.42)

donde

{3= k _

Lo!;} (tI - X o )2 En la forma de estado, estas ecuaciones quedan

YI =

¡

('YI - 'YY3 + VI)

o

(4.43)

Aún con un sistema de ecuaciones tan sencillo como el (4.43), se puede utilizar la computadora para determinar y graficar la solución. Una vez que se decIde usar este recurso, se hace evidente que no es más difícil

168 Sistemas electromecánicos SUBROUTINE FCT (X, Y, DERY, NDIM) REAL *4Y(NT)IM), DERY(NDIM), SOL(lOO,4) COMMON SOL,N,V,AM,AK,R,OL,B,C,A IF(Y(2» 1,1,2 DERY (1)=(V - Y(l)*R)*Cj A DERY (2)=0. DERY(3)=0. RETURN 2 DERY (1)=(V - Y(1)*R)*(Y(2)+C)j A+ Y(1)*Y(3)j(Y(2)+C) DERY (2)- Y(3) DERY (3)= - B*Y(3)j AM -AK*(Y(2)-OL)j AM -A*Y(1)*Y(1)j(2.*AM*(Y(2)+C)* 1(Y(2)+ C» RETURN END

FIGURA 4.11 Programa para la solución de un sistema no lineal. 6.0

t

4.0

2.0

OL-----~

20

____ ____ ____ ____- L_ _ _ _ 40 60 80 120 100 ~

~

~

~

¡(ms)_ (a)

0.3

t

0.2

E 2

20

40

80

60

100

120

¡(ms) ~

(b)

FIGURA 4.12 a) i(t) para una entrada escalón. b) x(t) para una entrada de voltaje escalón.

arcuitos eléctricos equivalentes 169

programar (4.41) que (4.43). Si se usa la ecuación no lineal, no hay que realizar ningún trabajo adicional para linealizar. Para analizar el relevador de la figura 4.9 se utiliza la subrutina RKGS y se grafican los resultados. La figura 4.11 muestra las ecuaciones programadas para el sistema no lineal. Estas ecuaciones se introducen a la RKGS mediante la subrutina exterior FeT. La figura 4.12 muestra la corriente y la posición para un voltaje escalón aplicado. Las constantes empleadas en este ejemplo son M = 10 g, b = 0.0001 N seg/cm, k = 1 N/cm, R = 1 n, X o = 2.13; en la figura 4.10 se mostraron el resto de los datos. 4.6 CIRCUITOS ELECTRICOS EQUIVALENTES Se sabe que la representación por circuito equivalente (como la estudiada antes de los transformadores) es sumamente útil para analizar un sistema y los sistemas electromecánicos pueden representarse mediante circuitos equivalentes solamente eléctricos. En el ejemplo que sigue, se ilustra el método para desarrollar un circuito equivalente de un sistema mecánico. Un sistema solamente mecánico está constituido por un resorte, una masa y un amortiguador, como se ilustra en la figura 4.13a, donde se supone que la fuerza de amortiguamiento es directamente proporcional a la velocidad. Si Fex t es una fuerza externa aplicada, la ecuación de equilibrio de fuerzas, de acuerdo con la ley de Newton es entonces

M.i+b.i+kx=FeXI Si se supone que la fuerza F ext es análoga al voltaje v, entonces el circuito de la figura 4.13 puede trazarse a partir de las siguientes correspondencias:

(a)

(b)

(e)

FIGURA 4.13 a) Un sistema mecánico. b) Circuito equivalente elédrico fuerza-voltaje. e) Circuito equivalente eléctrico fuerza-corriente.

170 Sistemas electromecánicos Analogía fuerza-voltaje FuerzaF Velocidad Amortiguación b Masa M Constante de resorte k

Voltaje v Corriente i Resistencia r Inductancia L Elastancia l/C = recíproca de la capacitancia

x

Si la fuerza se toma como análoga de la corriente, también se tiene la siguiente analogía, en base al principio de dualidad. Analogía fuerza-voltjae Voltaje v Corriente i Resistencia r Inductancia L Elastancia l/C = recíproca de la capacitancia

FuerzaF Velocidad Amortiguación b Masa M Constante de resorte k

x

Esto da como resultado el circuito que se muestra en las figuras 4.l3( b) Y (e).

En el ejemplo anterior se trató con un sistema puramente mecánico muy sencillo, pero el método también se aplica a sistemas electromecánicos más complicados. Con base en el análisis anterior, puede desarrollarse un circuito equivalente para el "relevador" que se estudió en la última sección. Para este propósito, se recordará la ecuación (4.42):

Si se hace

'YXl

di¡ . . L0dt +n¡ + YX¡ = v¡

(4.44)

Mi¡ +bx¡ + {3x¡- yi¡ =0

(4.45)

= V2, (4.44) puede escribirse como (4.46)

Sustituyendo 'YXl

= V2

en (4.45), se tiene

.. M. M x¡=-v2' y

. b bx¡ = -v2' y

Circuitos eléctricos equivalentes 171

+

VVv-~~----~----~-----+

FIGURA 4.14 Un circuito equivalente para el sistema electromecánico.

de tal suerte que (4.45) se hace

(4.47) donde C 2 = M /,,(2, L 2 = "(2 /{3 Y G 2 = b 1'Y 2 • Con el empleo de estas constantes, (4.46) y (4.47) pueden representarse por el circuito equivalente de la figura 4.14. Por otra parte, (4.44) y (4.45) pueden también escribirse como

(4.48)

(4.49) donde b = r 2 Y C 2 = 1/{3. Obsérvese que la reciprocidad de interacción mutua no es válida aquí, en virtud de que los coeficientes de los términos de acoplamiento en las dos ecuaciones no aparecen con el mismo signo. Esto puede tomarse en cuenta mediante el uso de un girador y el circuito equivalente se transforma entonces como se muestra en la figura 4.15.

G FIGURA 4.15 Un circuito equivalente alterno.

e,

172 Sistemas electromecánicos

4.7 SISTEMAS CON EXCITACION DOBLE y MULTIPLE Hasta aquí sólo se han considerado sistemas con una sola bobina (es decir, con entrada eléctrica única), pero los principios se aplican igualmente a sistemas de bobina múltiple (o sistemas multiexcitados). Para determinar la fuerza eléctrica, por ejemplo, es necesario determinar la energía almacenada. Así, para un sistema con doble excitación (o de dos bobinas) como el de la figura 4.16, pueden expresarse los eslabonamientos de flujo en términos de las inductancias de la manera siguiente: A) = L))i) + L 12 i2 A2 = Ll2i) + L 22 i2

Supóngase que existen cambios incrementales en los eslabonamientos de flujo, de tal manera que dA) = Llldi) + L 12 di 2 dA 2 = L 12 di)

+ L 22 di2

Para encontrar el cambio en la energía magnética almacenada, se multiplican las dA anteriores por las corrientes apropiadas. Por lo tanto dWm = i)dA) + i 2dA 2

La energía total almacenda se calcula integrando esta ecuación para obtener l L ·2 L .. + l L /·2 W m="2 11/)+ )2/)/2 "2 22 2

(4.50) .. + L 12/)/2 ., + L 22 1'2) ="2l (L ))/)·2 + L 121)/2 2

Además,( 4.50) puede generalizarse (4.51)

o, en notación matricial, };L"I Wm = -1 2

(4.52)

Problemas de valores instantáneos, medios y RCM 173

I i, (

L l1

\ f'J

L'2

i2 L 22

I FIGURA 4.16 Un sistema doblemente excitado.

donde i es una matriz columna, i es la transpuesta de i (es decir, i es una matriz fila) y L es la matriz de inductancia del sistema. Una vez que se determine la energía almacenada, el resto del procedimiento en el estudio de un sistema multiexcitado es semejante al ·lue se vio antes.

4.8 PROBLEMAS DE VALORES INSTANTANEOS, MEDIOS y RCM

Los valores instantáneo y promedio de una fuerza de origen eléctrico son importantes en el estudio de sistemas electromecánicos. Por ejemplo, para que se inicie el movimiento mecánico la fuerza instantánea no debe ser nula. En forma análoga, para un movimiento continuo, la fuerza promedio debe ser diferente de cero. La fuerza promedio se define como (4.53)

donde T es un lapso conveniente y F inst es la fuerza instantánea. De la definición de fuerza promedio surge una nueva interpretación de valor rcm (valor raíz cuadrático medio o eficaz), que se ilustra en las siguientes ecuaciones. Para un sistema inductivo excitado con corriente, la fuerza instantánea es (4.54)

174 Sistemas electromecánicos

donde la corriente i puede tener cualquier forma de onda. La fuerza promedio será, de (4.53) y (4.54), _ 1 Ji',rom-

aL

2' ax

1 (T' 2 T Jo 1 dt

(4.55)

Si al sistema lo excita una corriente directa [c.c., la fuerza promedio es F'

prom

=

1.2 aL [2 ax C.c.

(4.56)

El valor de la corriente directa que produce la misma fuerza promedio es, por lo tanto, de (4.55) Y (4.56),

[ =(.1 Jo c.c.

T

(T¡2dt){

(4.57)

Nótese que (4.57) coincide con la definición de valor rcm o eficaz. Así, con el objeto de que se ejerza la misma fuerza promedio para ambos tipos de excitación: c.c. y c.a., es necesario que el valor rcm o eficaz de la corriente alterna sea lineal a la magnitud de la corriente directa. En resumen, en este capítulo se han estudiado los principios fundamentales que gobiernan al proceso de conversión de energía electromecánica. Se aplicaron estos principios al estudio de la dinámica de diversos sistemas electromecánicos.

Problemas 4.1 El sistema electromecánico de excitación única que se muestra en la figura 4.17 está restringido a desplazarse sólo horizontalmente. Las dimensiones pertinentes se muestran en el diagrama. Determinar la fuerza eléctrica ejercida sobre el miembro móvil de hierro para a) excitación con corriente ¡ = 1 coswt b) excitación con voltaje

v= Vcoswt En ambos casos, no considerar la resistencia de devanado, los campos de fuga ni los efectos de borde. Se supondrá que toda la energía se almacena en los entrehierros de aire; es decir, que la permeabilidad del hierro es muy grande comparada con la del espacio libre. ¿Qué modificaciones habría que hacer si no se despreciara la resistencia de devanado?

Problemas de valores instantáneos, medios y RCM 175

w X w Sección transversal

R

+

Dirección posible de movimiento

v

fl,

/

Núcleo fijo

x

,

Elemento móvil

FIGURA 4.17 Problema 4.1 4.2 Una estructura electromagnética se caracteriza por las siguientes inductancias dependientes de 8: LII

=2+cos28=L22

L 12 = 1 +O.5cos8= L 21 Supóngase nula la resistencia de devanado. Encontrar el par (en función de 8) cuando ambos devanados estén conectados a la misma fuente, de tal manera que VI = V2 = 155 sen 377 t. 4.3 El sistema mostrado en la figura 4.18 tiene dos bobinas, con autoinducciones L 11 Y L 22 e inductancia mutua L 12. La bobina 1 lleva una corriente i 1 = 1 1 sen

l1

. . . -_ _ _----, ___x

I

I I

;, No, 1

o--L~"-1>\

0-'-;'2_-+----+-)2 No,2

L 22

FIGURA 4.18 Problema 4.3

I I I I

176 Sistemas electromecánicos WI t Y la 2 una i 2 = 12 sen W2 t. Las inductancias son LII = kl/x, L 22 = k2/X y L I2 = k3/~ siendo k¡, k 2 Y k3 constantes. Establecer una expresión para la fuerza instantánea en la armadura: Dar una expresión (en forma integral) para la fuerza promedio. Encontrar una relación entre W1 Y W2 para i) fuerza promedio máxima y ii) fuerza promedio mínima. Determinar los valores máximo, mínimo e intermedio de la fuerza promedio.

4.4 Un sistema mecánico está constituido por un resorte, masa y amortiguamiento, como se muestra en la figura 4.19. Seleccionar un sistema de coordenadas adecuado y establecer las ecuaciones de movimiento en base a las leyes de Newton. Obtener su circuito eléctrico equivalente usando a) La analogía fuerza-voltaje b) La analogía fuerza-corrien te

FIGURA 4.19 Problema 4.4 4.5 Los datos siguientes corresponden al sistema de la figura 4.20: rl, r2 = resistencias de los dos devanados; i 1, i2 = corrientes especificadas; ni, n2 número de vueltas; /10 = permeabilidad del espacio libre; A = área de la sección transversal del vástago central y de los de los extremos; M = masa de la armadura; k = constante del resorte b = coeficiente de rozamiento y lo = longitud del resorte cuando i = O. Suponer que toda la energía se almacena en el entrehierro de aire. Si i 1 = lc.c. e i 2 = 1m sen t, escribir

a)

A

o+

;,

" .A/lllt ?

,

v

;2

J

r2

JVvv

v2

r

n2

n,

A v

I

-

Armadura móvil. ¡;. = = Masa Núcleo fino ¡;. = =

A

J

Wff//,iff~Coeficiente de rozamiento

FIGURA 4.20 Problema 4.5

=b

Problemas de valores instantáneos, medios y RCM 177 i) Las ecuaciones eléctricas de movimiento en notación matricial. ii) La ecuación mecánica de movimiento. iii) Si las ecuaciones anteriores son no lineales, identificar los términos no li-

neales. b) Si i 1 = le.c. e i 2 = O obtener el punto de operación estable y linealizar la ecua-

ción alrededor de este punto. 4.6 Dos bobinas tienen sus autoinductancias e inductancia mutua (en Henries) como funciones del desplazamiento x (en metros) de la manera siguiente:

2

L¡¡=L22=3+ 3x;

Las resistencias son insignificantes. Ambas bobinas están excitadas por la misma fuente de voltaje VI = V2 = V = 100 cos 50 t volts. a) Encontrar una expresión para la fuerza eléctrica (esto es, la fuerza de origen

eléctrico ). b) Calcular el valor promedio en el tiempo de la fuerza en x = 1.

e) ¿Tiende la fuerza a incrementar o reducir a x? 4.7 En la figura 4.21 se muestra un sistema electromecánico. La estructura del núcleo es cilíndrica. En la misma figura se indica el resto de datos. La bobina tiene una resistencia insignificante y un voltaje ven sus terminales. a) Establecer las ecuaciones generales de movimiento eléctrica y mecánica. b) Dados a = 1 cm; lo = 5 cm; I1 = 4 cm; w = 0.1 cm y N = 100 vueltas, calcular la inductancia de la bobina para x = loe) Con los datos de b), determinar la fuerza en la tara para x = 2 cm e i = 2 A c.C.

N - vueltas r-l--+-_;...-----o+

v

Radio

=a

Cojinete cil índrico no magnético

FIGURA 4.21 Problema 4.7

178 Sistemas electromecá~icos

4.8 En un electroimán la relación i -A es

donde x es cierto entrehierro de aire arbitrario. Si A es la variable independiente, a) encontrar una expresión para la fuerza electromagnética en x = 0.5 Y en x = 1. b) ¿Se modifica la fuerza con x? Explicar.

4.9 El electroimán del Probl. 48 se excita con corriente continua de 3 A en una bobina de N vueltas. Si se requiere que el imán desarrolle una fuerza de 12 Nw en x = 1.0 cm, calcular N. 4.10 Para el sistema electromecánico de la figura 4.22, obtener una expresión para la fuerza eléctrica que se desarrolla. Establecer todas las hipótesis de simplificación que se hagan para esa obtención. Escribir las ecuaciones de movimiento eléctrica y mecánica. Encontrar la posición de equilibrio. ¿Cuál es la dirección de la fuerza eléctrica? Supóngase en seguida que se elimina el resorte y que no se toma en cuenta la fricción. Se aplica una fuerza externa para jalar al émbolo una distancia d. Determinar el cambio en la energía almacenada en el campo de acoplamiento. De haber un cambio en la energía almacenada ¿a dónde pasaría? O bien ¿de dónde provendría, según el caso? Sección transversal del núcleo w X w

J.I.:::::OO

+

Cojinete no magnético ---;--1' = 1'0' Coeficiente de rozamiento b

N = Vueltas

v

I ~x

x =O

FIGURA 4.22 Problema 4.10

4.11 La sección transversal del circuito magnético de la figura 4.23 es a. La tapa corrediza soporta una fuerza de amortiguamiento viscoso F frie = bx que se opone a su movimiento y que actúa en el soporte. La posición de x igual a Xo se obtiene sin excitación eléctrica. Con un v = Vm cos cot y sin tomar en cuenta la resistencia de devanado, determinar 10 que sigue, en términos de los símbolos dados: a) La fúerza magnética que actúa sobre la tapa corrediza

Problemas de valores instantáneos, medios y RCM 179

r

x r---------------~

N

Circuito magnético

+

v

FIGURA 4.23 Problema 4.11 La distancia x en función del tiempo en estado estacionario, así como la validez de esta expresión. e) La potencia promedio suministrada por la fuente de voltaje si el material magnético carece de pérdidas. b)

4.12 Para el sistema mostrado en la figura 4.9, con los parámetros definidos en el texto, (Sec. 4.5.5), determinar la variación de flujo en función del tiempo para una corriente de entrada de 2 A.

Capítulo 5 La máquina con conmutador

para C.C. En este capítulo se introduce un grupo de dispositivos electromagnéticos rotatorios: la máquina con conmutador para c.c. Al analizar este tipo de máquinas se emplearán muchos de los principios que se utilizaron en capítulos anteriores, en especial los conceptos de circuito magnético del capítulo 2 y las relaciones de fuerzas vistas en el capítulo 4. El material que se estudia en este capítulo generalmente se conoce sólo como "máquinas de c.c,". Se ha utilizado la palabra conmutador por varias razones. En primer lugar, el conmutador es la característica distintiva de los dispositivos que se discuten aquí. Sin él, estas máquinas no se diferenciarían de muchos otros tipos de máquinas. El conmutador es un rectificador/inversor mecánico que posibilita la conexión con una fuente de c.c. y, durate el proceso, otorga a esta configuración de máquina algunas de las más útiles características, como motor, generador y un dispositivo de control. En segundo lugar, el conmutador no es el tipo único de máquina de c.c. De hecho, no es del todo verdaderamente una máquina de c.c., si por "c.c." se entiende un dispositivo cuyas corrientes y voltajes son unidireccionales bajo una condición dada de velocidad y par. Los lectores con tendencia a la electrónica objetarán probablemente la inclusión de un rectificador/ inversor "mecánico" como parte de un dispositivo que se usa tan a menudo en sistemas electrónicos y de control, y preguntará: ¿por qué no realizar la función de rectificación/inversión con un dispositivo de estado sólido? Tales dispositivos son bastante conocidos actualmente y se les conoce como máquinas de c.c. sin escobillas. Si aún se tiene curiosidad por saber qué es la verdadera máquina de c.c., se debe tener presente que se trata de un dispositivo conocido como máquina homopolar, derivado del generador de disco de Faraday, desarrollado por Michael Faraday' en los años 30 del 181

182 La máquina con conmutador para

e.e.

siglo pasado. En varias aplicaciones aeroespaciales se ha usado una versión en metal líquido de esta configuración. Por consiguiente, se utiliza aquí la denominación "máquinas con conmutador para c.c." con el objeto de tipificar una configuración específica de máquinas de c.c. El término c.c. se emplea para describir esta clase genérica de máquinas e indicar que están energizadas convencionalmente por una fuente de energía eléctrica en c.c. Hay varios tipos de máquinas con conmutadores que operan normalmente con fuente de c.a.: incluyen motores de repulsión y máquinas polifásicas con conmutador. Estas últimas se usan en Europa para aplicaciones que en los EE.UU. utilizan máquinas con conmutador en c.c. Para finalizar, debe mencionarse que existe un tipo muy común de motor con conmutador en C.C., conocido como motor universal, que opera con fuentes de baja frecuencia en c.a. La máquina con conmutador de c.c. es un dispositivo rotatorio muy versátil y se construye en un rango amplio de dimensiones, desde dispositivos de control muy pequeños para potencias nominales del orden de 1 W hasta motores de grl\ndes dimensiones para potencias de 10,000 c.f. (HP) o mayores, para aplicaciones en molinos de laminado. Su uso principal es actualmente como motores industriales de impulsión, especialmente donde se requiere un par de gran magnitud controlado con toda precisión. Motores de este tipo de emplean en molinos de laminación para acero y alumnio, en motores de tracción, en grúas de construcción y móviles, trenes eléctricos, vehículos eléctricos y C~,J.TOS para golf. Las máquinas con conmutador se usan en herramientas p,)rtátiles alimentadas con baterías, en vehículos automotrices en calidad (le motores de arranque, en motores para ventiladores y en muchas aplicaciones de control, como activado res y dispositivos sensores de velocidad o de posición. Casi no existen usos modernos de máquinas con conmutador como generadores de potencia, aunque en las etapas iniciales de distribución de potencia electrica el sistema de tres hilos de Edison y el generador con conmutador de c.c. fueron los medios principales para suministrar energía eléctrica a usuarios domésticos e industriales. Aún quedan vestigios de estos sistemas en las zonas céntricas de las grandes ciudades. Sin embargo, debe notarse que la máquina con conmutador en c.c. es un dispositivo bilateral y que muchos motores con conmutador de c.c. operan a menudo como generadores en modo "regenerativo" o de "frenado dinámico", especialmente en aplicaciones de tracción. 5.1 DESCRIPCION DE UNA MAQUINA CON CONMUTADOR EN C.C.

Las figuras 5.1 a 5.4 muestran ejemplos de este tipo de máquinas. Físicamente, constan de un miembro rotatorio, llamado rotor o armadura, y de uno estacionario, denominado estator. Existen dos tipos de geometría comunes a todas las clases de máquinas rotarorias y que se designan por la

Descripción de una máquina con conmutador en C. C. 183

FIGURA 5.1 Vista en corte de un motor de 4 000 Hp Y 700 V (Cortesía de General Electric Company).

2) l.o,poltestllrminelesNdlman·

sloneron par. co08xl6o rápida y

Motores de tracción "JB" General Electric 6) LOI Illteme, de ¡)o'ta'allCobtlIas M optimizan para condlero·

.. .ligeros _ de larga vida • funcionamiento versátil

n"d8valocldad,corrlente,voltaja y C81'111, proPorcionando unS SUI-

tltllclón,Implade81CObllluyuna

Mavo,eXllectatlvadevldeutll. • #<

1) Coraza de aca,o magne' tleo P8,a al marCO del mo'

tor,8copladoconunoKtre" mo corrugado da unapiua. Las tapas s& maqulnaron contenu,a.pareasegu,.' una alln&aclon eKscta V pro' porclona, soporte. dgldol pa.alolcojlnel.l.

5)!.olconmutedor8s.conconl"true· ción de "10rduu

da a,o y acoplados con cobre plataado y mica "premlum" ..I.cclo· nada, o

(6.42)

donde _ _\ Po CPo-cos v: / oo

(6.43)

2. Prueba con rotor bloqueado: en esta prueba, se bloquea el rotor de la máquina (8 = 1) Y se le aplica un voltaje reducido, de tal suerte que flu-

ya la corriente nominal a través de los devanados de estator. Se registran la potencia, voltaje y corriente de entrada y se reducen a valores por fase, que se designan con Ps , V s e ls' respectivamente. En esta prueba se suponen insignificantes las pérdidas por hierro y no considera la rama paralelo de la figura 6.19. Los parámetros se determinan así: 2

Ps

, =,\+a '2=/2 (!

(6.44)

s

(6.45)

El circuito a partir de datos experimentales 281

donde s =cos

_) Ps VI

(6.46)

s s

En (6.44) Y (645), a es constante y es análoga a la razón de transformación de un transformador. Toma en cuenta el efecto de resistencia y reactancia de rotor referidas al estator, como se vio en la sección 6.5. Las pruebas descritas aquí son aproximadas, pudiendo afinarse. En las referencias 1 y 4 se encuentran detalles al respecto. La resistencia por fase'l de estator se puede medir directamente y conociendo re de (6.44) se puede determinar r~ = a 2 '2' resistencia de rotor referida al estator. No existe un método simple para determinar las reactancias de fuga XI = X 2 separadamente. El valor total de la reactancia de fuga se encuentra por (6.45) y, aproximadamente, puede considerarse que XI =X 2 • Se verá en seguida un ejemplo para ilustrar los cálculos involucrados para determinar las constantes de la máquina a partir de datos experimentales. Ejemplo 6.5

Los resultados de las pruebas sin carga y con rotor bloqueado en un motor de inducción trifásico conectado en Y son los siguientes:

Prueba sin carga

Prueba con rotor bloqueado

voltaje línea a línea potencia de entrada total = corriente de línea pérdidas por rozamiento y vendaval =

220 V lOOOW 20A 400W

V oltaje línea a línea

=

potencia de entrada total corriente de línea

=

30V 1500W 50A

Calcnlar los parámetros del circuito equivalente mostrado en la figura 6.19. V = 220 = 127 V

0V3

/0=20 A 1 Po= } (1000-400) =200 W

Así, de (6.41) a (6.43),

28'2 Máquinas de inducción

'm = ,¡,.

1272 200 =80.5 g

-

't'o-cos

-1

200 -860 20x 127 -

127

Xm = 20xO.99 =6.4g Ahora,

=~=17.32 V sV3

V

Is=50 A

p = 1500 =500 W s 3 De (6.44) a (6.46), 500

'e= 5Q2 =0.2 g ,¡,.

't's=cos

-1

500 540 17.32x50 =

Una vez conocidas las constantes del circuito, puede calcularse la operación de la máquina, como en el ejemplo 6.4.

6.8 CRITERIOS DE OPERACION DE LOS MOTORES DE INDUCCION Los dos ejemplos anteriores mostraron la utilidad del circuito equivalente y el método para determinar sus parámetros, a partir de datos experimentales, con el objeto de calcular la operación del motor. La operación de un motor de inducción puede caracterizarse por los siguientes factores:

1. 2. 3. 4. 5.

Eficiencia Factor de potencia Par de arranque Corriente de arranque Par máximo

Criterios de operación de los motores de inducción 283

Obsérvese que estas características se muestran en la figura 6.18. Al diseñar, deben incluirse las pérdidas por núcleo y las debidas a [2 r, así como los medios para la disipación de calor. No se pretende en este libro presentar una discusión detallada de los efectos de cambios de diseño -y consecuentemente de las variaciones de los parámetros- sobre cada característica de operación. Aquí se resumen los resultados en calidad de direcciones generales. Por ejemplo, la eficiencia es aproximadamente proporcional a (1 - s); de este modo, el motor sería compatible con una carga que corriera a la velocidad más alta posible. En virtud de que la eficiencia depende obviamente de las pérdidas P r, r~ y r) deben ser pequeñas para una carga dada. Para reducir las pérdidas por núcleo, la densidad de flujo operante (B), debe ser pequeña aunque esto impone un requerimiento conflictivo sobre la corriente de carga (1;) puesto que el par, determinado por la carga, depende del producto de B por [;. En otras palabras, un esfuerzo para reducir las pérdidas por núcleo más allá de un cierto límite, se traduce en un aumento de las pérdidas [2 r para una carga dada. Puede verse de los circuitos equivalentes (desarrollados en la sección 6.5) que puede mejorarse el factor de potencia reduciendo las reactancias de fuga y aumentando las magnetizantes. Sin embargo, no es prudente reducir las reactancias de fuga a un mínimo, pues estas reactancias restringf.n a un mínimo la corriente de arranque del motor. Se notan, otra vez, las ( ondiciones conflictivas entre un alto factor de potencia y una corriente de arranque baja. Además, el par máximo será más elevado con reactancias de fuga más reducidas. Un par de arranque elevado se produce con una r; grande; es decir, mientras más grande sea la resistencia de rotor, mayor será el par de arranque. Una resistencia r; grande se encuentra en conflicto con un requerimiento de alta eficiencia. Se puede llegar a las conclusiones anteriores considerando el circuito de rotor sólo como se muestra en el ejemplo que sigue. Ejemplo 6.6

Del circuito equivalente de rotor mostrado en la figura 6.12,a) encontrar r 2 para la cual el par sea máximo; b) ¿cuál es el deslizamiento en este par máximo? c) Determinar r 2 para un par de arranque máximo y d) ¿cuál es efecto de X 2 sobre el par? De la figura 6.12, la potencia desarrollada Pd por fase esta dada por

Pero la velocidad mecánica

Wm

está relacionada con la síncrona por

284 Máqui1llJS de inducción

Las dos ecuaciones anteriores proporcionan la expresión para el par electromagnético Te :

(6.47)

La corriente de rotor 12 la da (6.48)

De (6.47) Y (6.48) se tiene (6.49)

Para un Te máximo debe tenerse aTe/3r2 = 0, lo que, junto con (6.49), da

o sea

a)

y b)

En el arranque, s = 1 c)

d) Para una resistencia de rotor dada, el par de arranque será un máximo si X 2 :::: 0, según (6.49).

Control de velocidad en motores de inducción 285

Desde luego que el análisis anterior es sólo aproximado; no obstante, es factible llegar a conclusiones similares usando el circuito equivalente exacto como se indicó en el problema 6.5.

6.9 CONTROL DE VELOCIDAD EN MOTORES DE INDUCCION Debido a su sencillez y solidez, el motor de inducción tiene numerosas aplicaciones. Sin embargo, también tiene la desventaja de que su velocidad, en contraste con los motores de c.c., no puede variarse eficiente y continuamente dentro de un rango amplio de condiciones de operación. Se revisarán aquí sucintamente los posibles métodos para variar la velocidad del motor de inducción, ya sea en forma continua o discreta. Sale de los objetivos de este libro tratar todos estos métodos en detalle y el lector interesado puede consultar las referencias dadas al final de este capítulo. La velocidad del motor de inducción varía ya sea (1) modificando la velocidad síncrona del campo progresivo (2), cambiando el deslizamiento. Dado que la eficiencia del motor de inducción es aproximadamente proporcional a (1 - s), cualquier método de control de velocidad que dependa de la variación del deslizamiento ineficiente inherentemente. Por otra parte, si es constante la frecuencia de alimentación, variar la velocidad mediante cambios de la velocidad síncrona conduce sólo a cambios discretos en la velocidad del motor. Se tratarán estos métodos de control de velocidad con algún detalle.

6.9.1 Control de velocidad por cambios en la velocidad síncrona Recuérdese que la velocidad síncrona ns del campo progresivo de una máquina de inducción rotatoria está dada por

f

ns = 120p

donde p es el número de polos y f = frecuencia de alimentación, lo que indica que n2 puede modificarse ya sea (1) alterando el número de polos o bien (2) cambiando la frecuencia f. Ambos métodos se utilizan, por lo que aquí se consideran los detalles cualitativos pertinentes. 1. El método de modificación de polos. En este método, el devando de estator del motor se diseña de tal manera que, modificando las conexiones de las diversas bobinas (cuyas terminales se sacan), pueda cambiarse el número de polos del devanado en la razón 2 a 1, con lo que resultan dos velocidades síncronas. Se notará que sólo dos velocidades de operación

286 Máquinas de inducción

son factibles. Si se dispone de más devanados independientes (por ejemplo, dos) -cada uno dispuesto para modificación de polos- pueden obtenerse más velocidades síncronas (por ejemplo cuatro). Sin embargo, sólo pueden alcanzarse cambios discretos en la velocidad del motor mediante esta técnica. El método tiene la ventaja de que es eficiente y confiable, pues el motor tiene un rotor jaula de ardilla sin escobillas. Otro método de modificación de polos es por medio de modulación en amplitud de polo. Se tienen informes sobre motoresjauladeardiHa con devanado simple que dan tres velocidades de operación. Otro método, basado en cambio de polos, con tres o cinco velocidades, se conoce como "cambio de polo modulado en fase". Como el método más simple de cambio de polo, los de modulación de amplitud y de cambio de polo modulado en fase, dan variaciones discretas en la velocidad síncrona del motor.

2. El método de la frecuencia variable. Se insiste en señalar que la veJocidad síncrona es directamente proporcional a la frecuencia. Si es factible variar la frecuencia de alimentación, podrá también modificarse la velocidad síncrona del motor, en forma continua o discreta, según la forma de variación de aquélla. Sin embargo, el par máximo desarrollado por el motor es inversamente proporcional a la velocidad síncrona. Si se desea un par máximo constante, deben incrementarse el voltaje y frecuencia de alimentación, si se quiere aumentar la velocidad síncrona del motor. La dificultad inherente en la aplicación de este método 'es que la frecuencia de alimentación de que se dispone por lo común, es fija, por lo que el método sólo es aplicable cuando se disponga de alimentación en frecuencia variable. Con el advenimiento de los dispositivos de estado sólido, con capacidades de potencia comparativamente grandes, es posible ahora usar inversores estáticos para impulsar al motor de inducción. En el capítulo 8 se estudia con algún detalle el control par estado sólido de motores de inducción. 6.9.2 Control de velocidad por cambio en el deslizamiento Se comprenderá mejor el método de control de velocidad de un motor de inducción observando la figura 6.20. La curva punteada indica la característica velocidad-par de la carga. Las curvas en líneas llenas son las caracteristicas velocidad-par del motor de inducción bajo diversas condiciones (como resistencias de rotor diferentes: r~, r~', r;" o voltajes de estator diferentes: VI, V 2 ). Se dispone así de cuatro curvas distintas de velocidad-par y, por lo tanto, puede correr el motor en cualquiera de las cuatro velocidadesN¡, N 2 , N 3 Y N 4 para una carga dada. Obsérvese que a la derecha del par pico se localiza la región de operación estable del motor. En la práctica, puede cambiarse el deslizamiento del motor por uno cualquiera de los métodos siguientes.

Control de velocidad en motores de inducción 287

v, > 'í» Tí'

V2

> ,;"

t

Velocidad _ _ Región de operación estable para una máquina con voltaje de estator VI Y resistencia de

rotor r2'"

FIGURA 6.20 Control de velocidad cambiando el deslizamiento.

1. Método del voltaje de estator variable. Dado que el par electromágneti-

co es proporcional al cuadrado del voltaje aplicado, se obtienen diferentes curvas par-velocidad para distintos voltajes aplicados al motor. Para una resistencia de rotor r 2 dada, se muestran dos curvas de este tipo para dos voltajes aplicados VI y V 2 ; así, el motor puede correr a velocidades N 2 Ó N 4 • Si puede variarse continuamente el voltaje de VI a V 2 , la velocidad del motor puede también variar así entre N 2 y N 4 para una carga dada. Este método se aplica a los motores de inducción tipo jaula y a los de rotor devanado.

2. Método de la resistencia variable del rotor. Este método se aplica únicamente al motor con rotor devanado. En la figura 6.20 se muestra el efecto de insertar resistencias externas en el circuito de rotor para tres diferentes resistencias de rotor r~, r~' y r~". Para la carga dada son posibles tres velocidades de operación. Desde luego que para variación continua de la resistencia de rotor, es posible la variación continua de la velocidad. 3. Control mediante conmutación de estado sólido. Además del motor impulsado por inversor, puede controlarse la velocidad de motor de rotor devanado, insertando al inversor en el circuito de rotor, o controlando el voltaje de estator por medio de dispositivos de conmutación

288 Máquinas de inducción

de estado sólido, como rectificadores de silicio controlado (SCR o thyristores). La salida del SCR que alimenta al motor se controla ajustando su ángulo de disparo. El método para hacerlo es similar al de voltaje variable delineado antes. No obstante, se ha encontrado que el control mediante un SCR da un rango más amplio de operación y es más eficiente que otrosmetodosde control por deslizamiento (ver capítulo 8).

4. Control de velocidad mediante máquinas auxiliares. Existen otros diversos esquemas para controlar la velocidad del motor de inducción. Incluyen concatenación, el motor Schrage, el control Kramer, el Scherbius, etc., descritos con algún detalle en la referencias 1 y 4. En síntesis, debe señalarse que el método para regular la velocidad de un motor de inducción controlando su deslizamiento es básicamente ineficiente y puede tener además otras desventajas. El método basado en el control de la velocidad síncrona es eficiente, aunque difícil de llevar a la práctica, especialmente con máquinas sin escobillas (o tipo jaula) para variación continua de velocidad. Por lo que se ve está aún por desarrollarse un método eficiente, económico y satisfactorio para el control de velocidad en motores sin escobillas.

6.10 ARRANQUE DE MOTORES DE INDUCCION En el capítulo anterior se estudiaron las condiciones de arranque de un motor de c.c. Para el arranque de uno de inducción se aplican condiciones en cierta manera semejantes. Para acelerar el motor desde reposo hasta la velocidad de operación en régimen estacionario, la entrada de energía debe ser mayor que la energía cinética del motor y carga, cuando menos en la cantidad de las pérdidas totales. La corriente de entrada no debe ser excesiva para que se pueda cumplir lo anterior. Por ejemplo, no debe exceder más de seis veces a la corriente en carga total. El par de arranque debe ser alrededor de 1.5 veces el de carga total. Considérese en primer término la limitación de corriente. Algunos de los métodos comunes para limitar la corriente de estator en el arranque son: 1. Arranque con voltaje reducido. En el arranque, se aplica un voltaje re-

ducido al estator que se incrementa hasta el valor nominal cuando el motor se encuentra en el 25% de su velocidad final. Este método tiene la limitación evidente de requerir una fuente de voltaje variable y además el par de arranque cae sustancialmente. El llamado método Y -delta de arranque es un caso de arranque en voltaje reducido. Si el estator está normalmente conectado en delta, la reconexión a Y reduce el voltaje de fase, generándose una corriente menor en el arranque. Por

Arranque de motores de inducción 289

FIGURA 6.21 Arranque Y-delta. Los conmutadores en W corresponden a la conexión Y y en D a la delta.

ejemplo, durante el arranque, si la corriente de línea es alrededor de cinco veces la corriente en carga total en un estator conectado en delta, la corriente en la conexión Y será menor que dos veces el valor en carga total. Sin embargo, al mismo tiempo, el par de arranque para una conexión Y sería alrdedor de un tercio de su valor en conexión delta. La ventaja del arranque Y-delta es que no es caro y sólo requiere un conmutador de tres polos (o tres polos simples) doble tiro (o conmutadores de este tipo), como se muestra en la figura 6.21.

2. Limitación de corriente por resistencia en serie. Se usan resistencias en serie, introducidas en las tres líneas algunas veces para limitar la corriente de arranque. Estas resistencias se cortocircuitan una vez que el motor gana velocidad. Este método tiene la desventaja obvia de ser ineficiente, a causa de las pérdidas adicionales en las resistencias externas. T

n

= ns

Velocidad

FIGURA 6.22 Efecto de modificar la resistencia de rotor sobre el arranque de un motor con rotor devanado.

290 Máquinas de inducción

Con referencia nuevamente al par de arranque, se señala otra vez que depende de la resistencia de rotor, como se vio en la última sección. De este modo, una resistencia de rotor alta, se traduce en un par de arranque elevado. Por lo tanto, en un máquina con rotor devanado (ver figura 6.22), puede usarse convenientemente la resistencia externa en el circuito de rotor con un rotor de barra profunda, donde la profundidad de la ranura es dos o tres veces más grande que su anchura (ver figura 6.23). Las barras de rotor alojadas en ranuras profundas proporcionan una resistencia efectiva alta y un par grande en el arranque. Bajo condiciones normales de operación, con deslizamientos bajos, la resistencia del rotor, sin embargo, llega a ser más baja y más alta la eficiencia. Esta característica de la resistencia de barra de rotor es una consecuencia del efecto pelicular. En virtud de este efecto, la corriente tiende a concentrarse en la cima de las barras durante el arranque, cuando la frecuencia de las corrientes de rotor es alta. En este punto, la frecuencia de las corrientes de rotor será la misma que la de entrada de estator (por ejemplo, 60 Hz). En operación normal, la frecuencia de las corrientes de rotor (= frecuencia de deslizamiento = 3 Hz en % 5 de deslizamiento y 60 Hz,es mucho menor. A este nivel de operación, el efecto pelicular es despreciable, y la corriente se distribuye casi uniformemente en toda la sección transversal de la barra. El efecto peculiar se usa en forma alterna en un rotor de doble jaula (figura 6.24), donde la jaula interior está pro fundamente alojada en el hierro y tiene barras de baja resistencia. En el arranque, a causa del efecto pelicular, domina la influencia de la jaula exterior produciéndose así un par elevado de arranque. Mientras se acelara, la corriente penetra a profundidad total dentro de la jaula inferior -por no haber casi efecto pelicular- lo que da lugar a una operación eficiente en régimen estacionario. Obsérvese que bajo condiciones de velocidad normal, ambas jaulas llevan corrientes, incrementando así un poco la capacidad del motor. 6.11 MOTORES DE INDUCCION MONOFASICOS

En secciones anteriores se consideraron el motor de inducción polifásico -preferentemente el trifásico- operando bajo condiciones balanceadas. Consideremos ahora un motor de inducción trifásico corriendo con carga ligera. Si se desconecta una de las líneas de alimentación, el motor continúa corriendo, aunque a velocidad diferente. Una operación tal de un motor de inducción trifásico puede considerarse como operación de un motor monofásico. Considérese ahora el motor trifásico en reposo y alimentado por una fuente monofásica. Obviamente, el motor no arrancará porque se tiene un campo magnético pulsante en el entre hierro de aire, en vez de uno magné-

Motores de inducción monofásicos 291

(a)

(b)

FIGURA 6.23 a) Ranuras abiertas. b) Ranuras parcialmente cerradas.

FIGURA 6.24 Forma de ranuras para un motor de doble jaula.

tico rotatorio, que es el que se requiere para la producción del par, como se analizó antes. Se concluye así que un motor de inducción monofásico no arranca por sí mismo, pero que si se arranca por algún medio continúa girando, lo que implica que para que arranque por sí mismo debe dotársele de medios auxiliares de arranque. En una sección posterior se examinarán los diferentes medios de arranque del motor de inducción monofásico. Sin considerar el mecanismo de arranque, la diferencia esencial entre el motor de inducción trifásico y el monofásico radica en que éste tiene un devanado de estator único que produce un campo de entrehierro de aire estacionario en el espacio, pero alterno en el tiempo. El trifásico posee un devanado de tres fases que produce un campo magnético rotatorio invariante en el tiempo dentro del entrehierro de aire. El rotor del motor de inducción monofásico es casi siempre de tipo jaula, semejante al de un polifásico. La capacidad de un monofásico de las mismas dimensiones que uno trifásico es menor, como era de esperarse y por lo general se especifica como motor de potencia fraccionaria. Los motores monofásicos se usan profundamente en utensilios del hogar, en ventiladores, etc. 6.11.1 Análisis de operación de motores de inducción monofásicos

De lo que se ha expuesto antes, se insistirá en señalar que el campo magnético producido por el estatorde un motor monofásico es alterno en el tiempo. El campo induce una corriente y, consecuentemente, una fmm en el circuito de rotor y gira con el rotor. Puede analizarse un motor de inducción monofásico considerando las fmms, los flujos, voltajes inducidos (tanto por rotación como por efecto de transformador) y las corrienteS' producidas separadamente por el estator y por el rotor. Un método así conduce a la

292 Máquinas de inducción

teoría de campo cruzado. Empero, es posible también analizar el motor monofásico de una manera semejante a la que se utilizó para el polifásico. Se insistirá en que este último opera sobre la base de la existencia de un campo magnético rotatorio. Este método se funda en el concepto de que un campo magnético alterno equivale a dos campos magnéticos rotatorios que giran en direcciones opuestas. Cuando este concepto se expresa matemáticamente, el campo alterno es de la forma

B( O, t) = Bm cos Osenwt

(6.50)

La ecuación (6.50) puede entonces reescribirse como

1

+"2 Bmsen(wt+ O)

(6.51 )

En (6.51), el primer término del segundo miembro es un campo magnético progresivo y el segundo término corresponde a uno regresivo. La teoría basada en una resolución de un campo magnético alterno en dos campos rotatorios encontrados u opuestos se conoce como teoría del campo de doble revolución. La dirección d.: rotación del campo progresivo se supone coincidente con la del rotor. Df' esta manera, si el rotor gira a n rpm y ns es la velocidad síncrona en rpm, f'i deslizamiento sr del rotor respecto al campo rotatorio progresivo coincide con s, definido por (6.10), o sea: ns-n n s1 = s = - - = 1- ns ns

Pero el deslizamiento da

Sb

del rotor respecto al flujo rotatorio regresivo lo

ns -( - n) Sb

(6.52)

=

ns

n

=

1+ - = 2 - s ns

(6.53)

Se conoce de la operación de motores polifásicos que n < n., (6.52) corresponde a operación motor y (6.53) denota la región de frenado. De este modo, los dos pares resultantes tienen una influencia opuesta sobre el rotor. La relación de par para el motor de inducción polifásico es aplicable a cada uno de los dos campos magnéticos rotatorios del monofásico. ASÍ, el par resultante de un motor de inducción monofásico puede escribirse como

Motores de inducción monofásicos 293

li

(1-s)

li

(1-s)

T =----r -----r e Wm S 2 W m (2 - S) 2

(6.54)

Se observa ahora de (6.51) que la amplitud de los campos rotatorios es un medio del flujo alterno. Las reactancias de fuga y magnetizante del motor pueden así dividirse equitativamente para corresponder a los campos rotatorios progresivos y regresivo. En la figura 6.25a se muestra el circuito equivalente aproximado de un motor de inducción monofásico, basado en la teoría de campo de doble revolución. En la 6.25b se muestran las características par-velocidad en forma cualitativa. El ejemplo que sigue ilustra la utilidad de ese circuito. Ejemplo 6.7

Con referencia a la figura 6.25a, las constantes de un motor monofásico de 1/4 hp, 230 V, cuatro polos, 60 Hz son: rl = 10.0 n, XI = 12.8 n = X2 Y x m = 258.0 n. Para un voltaje aplicado de 210 V, con deslizamiento del 3%, calcular: a) la corriente de entrada, b) el factor de potencia; e) la potencia desarrollada; d) la potencia en la flecha (si las pérdidas mecánicas son de 7 W) y e) la eficiencia (si las pérdidas en el hierro son de 35.5 W en 210 V). Para el circuito dado y con los datos proporcionados, se tiene 0.5r2= S

0.5r2

11.65 =194.16Q 2xO.03 11.65

2-s = 2(2-0.03) =2.96Q y

jO.5xm = j129Q jO.5x2 =jO.5x, =j6.4Q

Para el circuito de campo progresivo 194.16Xj129 . Zf= 194.16+jI29 =59.2+}86

y para el de campo regresivo 2.96Xj129 Zb= 2.96+jI29 ;;;;;;2.96

La impedancia total serie Ze es Ze = Z, + Zf+ Zb =(10+ jI2.8) + (59.2+ j89) + 2.96= 124L 55°

294 Máquinas de inducción x,

/,

+

0.5 r2 s

v 0.5 r 2 (2 - s) O·5x m

(a)

Par por campo positivo /-

~

/

C1I

n.

'\

/ /

\

\

,/

/

/'

\ \

-- --

/ " / " Par por campo negativo

/

,_ ......

/ /

(b)

FIGURA 6.25 a) Circuito equivalente para un motor monofásico, basado en la teoría del campo de revolución. b) Características par velocidad de un motor de inducción monofásico, basadas en la teoría del campo de revolución.

Motores de inducción monofásicos 295

1. Corriente de entrada /=

V

Ze =

2. F¡¡.ctor de potencia cos 55°

210 o 124L55 0 = 1.7 L -55 A

= 0.573 (atraso)

3. Potencia desarrollada

0.5r2 0.5r P = _ _ /2- _ _2 /2 ) (l-s) ( s f d 2-s b

ya que s = 0.03 (pequeño). Pero V, = /Z, = 1.7 (59.2 + j89) = 182 V Y V b = IZ b = 1.7 X 2.96 = 5.04 V. O sea 1822

5.042 )

Pd = ( 194 - 2.96 (1-0.03)= 156 W

4. Potencia en la flecha Ps = P d - Prot = 156 - 7 = 149 W 5. Potencia de entrada VI cos e = 210 X 1.7 X 0.573 = 204 W potencia 113.5 de salidaPs - Ph = 149 .. 35.5 = 113.5 W eficiencia - - = 55.6%. 204

6.11.2 Arranque de motores monofásicos Se sabe ya que por carecer de campo magnético rotatorio, cuando el rotor de un motor monofásico está en reposo, no es posible que arranque por sí mismo. Los dos métodos para arrancar un motor monofásico son: introducir un conmutador y escobillas, como en un motor de repulsión, o bien producir un campo rotatorio por medio de un devando auxiliar, como un divisor de fase. A continuación se analizará este último método. De la teoría del motor polifásico, se sabe que para tener un campo magnético rotatorio debe disponerse de dos fmms cuando menos, desplazada una de la otra en el espacio y con corrientes que difieran en fase temporal. Así, en un motor monofásico, un devando de arranque en el estator se toma como segunda fuente de fmm. La primera de ellas proviene del devanado principal de estator. Enseguida se resumen los diversos métodos para alcanzar los corrimientos en fases espacial y temporal entre las fmms de devanado principal y de devanado de arranque. 1. Motores con fase dividida. En la figura 6.26a se representa esquemáticamente este tipo de motor, donde el devanado principal tiene una re-

296 Máquinas de inducción

sistencia relativamente baja y una alta reactancia. El devando de arranque, sin embargo, posee una alta resistencia y una baja reactancia y tiene un interruptor centrífugo, como se indica. El ángulo de fase O' entre las dos corrientes I m e Is es de 30 a 45°. El par de arranque T s lo da (6.55)

donde K es una constante. Cuando el rotor alcanza una cierta velocidad (alrededor del 75% de su velocidad final) entra en acción el interruptor centrífugo y desconecta del circuito al devanado de arranque. La característica par-velocidad del motor con fase dividida es de la forma mostrada en la figura 6.26b. Motores de este tipo se utilizan en, ventiladores, sopladores, etc. y se especifican para potencias hasta de 1/2 hp. Es posible desarrollar un par de arranque fuerte por un motor con fase dividida, insertando una resistencia en serie en el devando de arranque. Un efecto en cierta manera similar puede obtenerse insertando una reactancia inductiva en serie en el devando principal. Esta reactancia se cortocircuita cuando el motor desarrolla su velocidad de operación. 2. Motores con capacitor de arranque. Es posible aumentar el ángulo O' en (6.55) conectando en serie un condensador con el devanado de arranque, como se muestra en la figura 6.27, con lo que el motor desarrollará un par de arranque elevado. Motores de este tipo no se limita a capacidades de potencia fraccionarias, pudiendo alcanzar hasta 10 hp. En 100 V, un motor de un caballo requiere un condensador de 400 J.l F, aproximadamente, mientras que uno de 70 J.lF basta para un motor de 1/8 de caballo. Los capacitores que se usan generalmente son electrolíticos de bajo costo y pueden proporcionar un par de arranque que es casi cuatro veces el nominal.

s

=

s = O

1 ~

(a)

Deslizamiento (b)

FIGURA 6.26 a) Conexiones para un motor de fase dividida. b) Una característica par-velocidad.

Motores de inducción monofásicos 297

'""''"' "' '1 e Devanado de arranque

Rotor tipo jaula

principal

FIGURA 6.27 Un motor con capacitor de arranque.

Como se ilustra en la figura 6.27, el capacitar es simplemente una ayuda para el arranque y se desconecta por el interruptor centrífugo cuando el motor alcanza una velocidad predeterminada. No obstante, algunos motores carecen del interruptor centrífugo; en éstos, se tienen el devanado de arranque y el capacitar como elementos para operación permanente y los capacitares son muchos menores. Por ejemplo, un motor de 100 V, 1/2 hp requiere una capacidad de 15 pF. Una tercera clase de motores con capacitar emplea dos capacitares: uno que se deja permanentemente en el circuito junto con el devanado de arranque y el otro que se desconecta por el interruptor centrífugo. Estos motores son, de hecho, motores de inducción bifásicos desbalanceados. 3. Motores de polo sombreado. Otro método para arrancar motores de inducción monofásicos muy pequeños es mediante el uso de una banda sombreante sobre los polos, como se muestra en la figura 6.28, donde el devanado principal monofásico se arrolla también sobre los polos salientes. La banda sombreante es simplemente una tira de cobre cortocircuitada arrollada sobre una porción del polo. Un motor de este tipo se conoce como motor de polo sombreado. El objeto de la banda sombreante es retardar la parte del flujo que pasa por ella en relación al flujo que emerge del resto de la cara del polo. De esta manera, el flujo en la parte sin sombra alcanza su máximo antes que el ubicado dentro de la parte con sombra. Se tiene así un desplazamiento progresivo del flujo desde la dirección de la parte sin sombra hacia la sombreada del polo, como se muestra en la figura 6.28. El efecto del desplazamiento progresivo del flujo es semejante al de un flujo rotatorio y a causa de ello la banda sombreante proporciona un par de arranque. Los motores con polo sombreado son los menos

298 Máquinas de inducción

+

v

Rotor tipo jaula

Espira de ,.."........,- sombra

FIGURA 6.28 Un motor con polo sombreado.

costosos de los motores de caballos de potencia fraccionarios y se especifican hasta 1/20 de hp. En resumen, en este capítulo se estudiaron las características de régimen estacionario de motores de inducción polifásicos y monofásicos, operando bajo condiciones balanceadas. En el capítulo 9 se tratan temas adicionales relacionados con el motor de inducción, como su dinámica y operaciones des balanceadas. En el 8 se introducen el control con estado sólido de motores de inducción.

Bibliografía 1. A. S. Langsdorf, Theory of A lternating Current Machinery, McGraw-Hill Book Company, Nueva York, 1955. 2. D.C. White, and H.H. Woodson, Electromechanical Energy Conversion, John Wiley & Sons, Inc. Nueva York, 1959. 3. S. Seely, Electromechanical Energy Conversion, McGraw-Hill Book Company, Nueva York, 1962 4. A.F. Puchstein, T.C. Lloyd, and A.G. Conrad, Alternating-current Machines, Tercera Edición, John Wiley & Sons, Inc., Nueva York, 1954.

Problemas 6.1. Un motor de inducción de cuatro polos, 60 Hz, corre en 1710 rpm. Calcular: a) el deslizamiento por ciento, b) la frecuencia de las corrientes de rotor y c), la velocidad de campo magnético rotatorio producido por i) el estator y ii) el rotor, respecto al estator, en rpm y en radfseg. 6.2. Un motor de inducción con rotor devanado, de dos polos, 60 Hz, tiene 120 V por fase en las terminales del estator. El voltaje inducido es de 3.81 V por fase.

Problemas 299

6.3.

6.4

6.5. 6.6.

6.7.

Si se supone que el estator y el rotor tienen iguales números efectivos de vueltas por fase, calcular: a) la velocidad del motor y b), el deslizamiento. Un motor de inducción tipo jaula está constituido por 42 barras, cada una con una resistencia de 4.12 X 10-5 Ohms (incluyendo la resistencia de los dos anillos terminales). El devanado de estator tiene los datos siguientes: seis polos, trifásico, 36 ranuras, 144 vueltas por fase, y un factor de devanado de 0.836. Calcular la resistencia de rotor equivalente por fase que se refiera al estator. La potencia que cruza el entrehierro de aire de un motor de inducción es de 24.3 kW. Si la potencia electromagnética desarrollada es 21.9 kW, ¿cuál es el deslizamiento. La pérdida rotacional en este deslizamiento es 350 W. Calcular el par de salida si la velocidad síncrona es de 3600 rpm. Obtener una expresión para el deslizamiento, utilizando la figura 6.17a, en el que el motor desarrolle el par máximo. Establecer una expresión para este par. Un motor de inducción trifásico, 230 V, 60 Hz, de dos polos conectado en Y, opera con un deslizamiento del 3% mientras toma una corriente de línea de 22 A. La resistencia de estator y la reactancia de fuga por fase son 0.1 y 0.2 Ohms, respectivamente. La reactancia de fuga del rotor es de 0.15 Ohms. Calcular: a) la resistencia de rotor, b) la potencia que cruza el entrehierro de aire y e) la potencia desarrollada. No considerar a X m • Las constantes por fase de un motor de inducción, conectado en Y, trifásico, cuatro polos 600 v, 60 Hz, con rotor devanado son: r¡ = 0.75 n r2

6.8.

6.9.

6.10.

6.11.

6.12.

= 0.80n

X¡=X 2 =2.0n X m =50.0n

No se consideren las pérdidas por núcleo. a) Calcular el deslizamiento para el que tiene lugar el par máximo desarrollado, b) encontrar el valor del par máximo; e) especificar el rango de velocidad para una operación estable del motor y d), calcular el par de arranque y compararlo con el máximo. Repetir las partes a) y b) del problema 6.7, considerando sólo el circuito de rotor. Suponer r¡ = X¡ = O, en cuyo caso pueden tomarse 600 V como el voltaje de línea aplicado al rotor. Un motor conectado en Y, dos polos, 25 Hz, 440 V, tiene una reactancia magnetizante de 10.5 Ohms por fase y una reactancia de fuga de rotor de 0.12 Ohms por fase. Usando sólo el circuito de rotor, determinar el deslizamiento para un par electromagnético máximo y una resistencia de fuga de rotor por fase de a) 0.03 Ohms, b) 0.06 Ohms y e) 0.1 Ohms. De estos resultados, mostrar el efecto de la resistencia de rotor sobre las características par-velocidad del motor. No tomar en cuenta a rl Y XI' Para el motor del problema 6.7, determinar el valor por fase de la resistencia que deba conectarse en el circuito de rotor para obtener el par máximo del motor al arrancar. Esbozar cualitativamente las características par-velocidad de un motor de inducción, comparándolas con las normales y mostrando los efectos de lo que sigue: a) variación de la frecuencia, conservando constante el voltaje aplicado; b) variación del voltaje aplicado, conservando constante la frecuencia. Un motor de inducción, conectado en Y, de cuatro polos, 60 Hz, trifásico, 220

300 Máquinas de inducción V, tiene una resistencia de estator por fase de 0.25 Ohms. Las pruebas sin carga y de rotor bloqueado dieron los datos que siguen para este motor:

voltaje de estator en prueba sin carga = 220 V corriente de entrada = 3.0 A potencia de entrada = 600 W pérdida por razonamiento y vendaval =300 W Prueba de rotor bloqueado: voltaje de estator = 34.6 V corriente de entrada = 15.0 A potencia de entrada = 720 W a) Obtener el circuito equivalente aproximado para la máquina. b) Si la máquina opera como motor con un 5% de deslizamiento, calcular la potencia desarrollada, el par desarrollado y la eficiencia. e) Determinar el deslizamiento para el que ocurre el par máximo y calcular el valor de éste.

Capítulo 7

Máquinas , slncronas

Las máquinas síncronas se encuentran entre los tres tipos más cm J.unes de máquinas eléctricas. Las otras dos, la máquina con conmutado' en C.c. y la máquina de inducción polifásica, se consideraron en los capít 110s precedentes. La mayor parte de la potencia eléctrica para el uso diario se produce mediante geb.eradores síncronos polifásicos, que son las máquinas eléctricas más grandes de una sola unidad que se fabrican. Por ejemplo, son muy comunes los generadores síncronos de algunos centenares de megavolts-ampere (MV A) y se espera que en los próximos años se produzcan algunos de miles de MV A. Estas máquinas se denominan síncronas porque operan a velocidades y frecuencias constantes bajo régimen estacionario. Como la mayoría de las máquinas rotatorias, una máquina síncrona es capaz de operar como motor o como generador. No obstante, ya que muy pocos impulsores industriales funcionan a velocidades fijas, rara vez encuentran las máquinas síncronas aplicaciones como motores de impulsión, en comparación con los motores de inducción o de c.c. Encuentran más bien una amplia aplicación como generadores, algunos de ellos operan en paralelo en estaciones de potencia, donde comparten la carga con cada uno de los otros y, en un tiempo dado, alguno puede no soportar carga. En un caso así, en lugar de apagarlo, se le permite "flotar" en la línea en calidad de motor síncrono sin carga. En una sección posterior se analizará una operación de este tipo de un motor síncrono. La operación de un generador síncrono se basa en la ley de Faraday de inducción electromagnética, y funciona de una manera muy semejante a uno de c.c., en el que la generación de fmms se realiza por el movimiento relativo de conductores y flujo magnético. Sin embargo, es obvio que un generador síncrono no posee un conmutador como el de c.c. Las dos par-

301

302 Máquinas sI'ncronas

tes fundamentales de una máquina síncrona son la estructura de campo magnético, con un devanado excitado con c.c., y la armadura, que frecuentemente cuenta con un devanado trifásico en el que se genera la fem alterna. Casi todas la máquina síncronas modernas tienen armaduras estacionarias y estructuras de campo rotatorias. El devanado de c.c. en la estructura de campo rotatoria se conecta a la fuente externa a través de anillos colectores y escobillas (véase nuevamente en el capítulo 6 la construcción del motor de inducción de tipo anillos colectores). Algunas estructuras de campo no disponen de escobillas, sino más bien de excitación sin escobillas, mediante diodos rotarorios. En algunos aspectos, el estator que lleva los devanados de armadura es similar al de un motor de inducción polifásico, estudiado en el último capítulo. En la sección siguiente se analizarán algunos de las características de construcción de máquinas síncronas. 7.1 CARACTERISTICAS DE CONSTRUCCION DE MAQUINAS SINCRONAS

Algunos de los factores que norman la construcción de la máquina síncrona son los siguientes: 1. Forma de excitación. La estructura de campo es por lo general el miembro rotatorio de una máquina síncrona y se alimenta con un devanado excitado con c.c. para producir el flujo magnético. La excitación de c.c. puede proporcionarse mediante un generador de c.c. autoexcitado que se monta en la misma flecha que el rotor de la máquina síncrona. A este generador se le conoce como el excitador. La corriente continua generada de esta manera alimenta al devanado de campo de la máquina síncrona como se muestra en la figura 7.1. En máquinas de baja velocidad con capacidades elevadas, como generadores hidroeléctricos, el excitador puede no ser autoexcitado, sino que con excitador piloto, autoexcitado o con imán permanente se activa al excitador (figura 7.7). Los problemas de mantenimiento de generadores de c.c. directamente acoplados imponen limitaciones a esta forma de excitación en capacidades de 100 MW. Otra forma de excitación se obtiene con diodos de silicio y thyristores, que no ofrecen problemas de excitación para máquinas síncronas grandes. Los dos tipos de sistemas de excitación con estado sólido son: a) Sistemas estáticos con diodos estacionarios o thyristores, en los que se alimenta la corriente al rotor a través de anillos colectores. b) Sistemas sin escobillas, con rectificadores montados en flecha y que giran con el rotor, evitando así la necesidad de escobillas y anillos colectores. La figura 7.2 muestra un sistema de excitación sin escobillas.

Construcción de máquinas síncronas 303

FIG URA 7.1 Vista en corte de una máquina síncrona de polos salientes (Cortesía de General Electric Company).

2. Estructura de campo y velocidad de máquina. Se mencionó ya que la máquina síncrona es una máquina de velocidad constante. Esta velocidad ns ' conocida como velocidad síncrona, la da (6.9). Se repite en seguida esta ecuación por convenir así: 1201 p

n=-s

(7.1)

De esta manera, una máquina síncrona de dos polos, 60 Hz, debe girar a 3600 rpm, mientras la velocidad síncrona de una de 12 polos, 60 Hz, es de sólo 600 rpm. La estructura de campo de rotor depende entonces de la velocidad de la máquina. Así, los turbogeneradores, que son máquinas de alta velocidad, tienen rotores redondos o cilíndricos (ver figuras 7.3 Y 7.4). Los generadores hidroeléctricos y díeseleléctricos son máquinas de baja velocidad y tienen rotores de polos salientes, como se ve en las figuras 7.5 y 7.6. Estos rotores son menos costosos en su fabricación que los

304 Máquinas síncronas

FIGURA 7.2 6 kV.

Rotor de un generador síncrono sin escobillas, de 3360 kV A Y

Flujo de campo

Devanado de campo

FIGURA 7.3

Devanado de campo en un rotor redondo.

Construcción de máquinas síncronas 305

FIGURA 7.4 Rotor a turbina con enfriamiento de agua directo, durante el montaje de los conductores huecos de amortiguamiento.

Flujo de campo

Devanado de campo

FIGURA 7.5

Devanado de campo en un rotor con salientes.

306 Máq umas . s(ncrolUlS

FIGURA MV A, 13.87 .6 RotorSla dede Brown una m' . síncrona de 152.5 Boveride compan;~uma kA (Corte' polos salientes

Construcción de máquinas s{ncronas 307

redondos. Los de polos salientes no son adecuados en máquinas de alta velocidad, a causa de las fuerzas centrífugas excesivas y esfuerzos mecánicos elevados que desarrollan en velocidades del orden de las 3600 rpm. Otro aspecto en la construcción de una máquina síncrona procede del montaje del rotor. Por ejemplo, una máquina impulsada por turbina con rotor redondo (figura 7.4) o una impulsora con diesel con rotor saliente (figura 7.1) tiene un rotor montado horizontalmente. Una máquina impulsada con rodete de agua (figura 7.7) tiene invariablemente un rotor de polo saliente, montado en forma vertical. 3. Estator. El estator de una máquina síncrona es semejante al de un motor de inducción polifásico (ver figura 7.8). No hay, esencialmente, diferencia entre el estator de una máquina de rotor redondo y uno de máquina con rotor saliente. Los estatores de los generadores hidroeléctricos, tienen, no obstante, por lo general una armadura de diámetro grande en comparación con otros tipos de generadores (figura 7.9). El núcleo de estator está formado por paquetes en laminados de alta calidad, con devanados imbricados alojados en ranuras. 4. Enfriamiento. Puesto que las máquinas síncronas se construyen en dimensiones extremadamente grandes, se diseñan para portar corrientes sumamente altas. Una densidad de corriente de armadura típica puede ser del orden de 10 A/mm en una máquina de buen diseño. Además, la carga magnética del núcleo es tal que se satura en muchas regiones. Las cargas eléctrica y magnética severas en una máquina síncrona producen calor, que debe disiparse convenientemente. De esta manera, la forma en que las partes activas de una máquina se enfrían, determina su estructura física global. Además de aire, algunos de los refrigerantes usados en máquinas síncronas incluyen agua, hidrógeno y helio. La figura 7.4 muestra un rotor a turbina con enfriamiento directo con agua, durante el montaje de los conductores huecos de amortiguamiento. En la 7.9 se muestra la disposición de enfriamiento del estator de un generador a turbina. 5. Barras de amortiguamiento. Hasta aquí sólo se ha hecho referencia a dos devanados de una máquina síncrona: el de armadura trifásico y el de campo, Se indicó, asimismo que en régimen estacionario, opera la máquina a velocidad constante, a la síncrona precisamente. No obstante, como cualquier otra máquina eléctrica, la síncrona experimenta transitorios durante el arranque y bajo condiciones anormales. Durante los transitorios, el rotor puede soportar oscilaciones mecánicas y su velocidad se aparta de la síncrona, lo que constituye un fenómeno indeseable. Para contrarrestar este fenómeno, se monta

308 Máquinas s{ncronas

FIGURA 7.7 Montaje de un rotor de un generador hidroeléctrico (Cortesía de Brown Boveri Company).

Construcción de máquinas síncronas 309

FIGURA 7.8 Proceso de montaje de conductores en las ranuras de un medio estator de una máquina síncrona (Cortesía de Brown Boveri Company).

310 Máquinas slncronas

FIGURA 7.9 Región de devanado terminal de un generador a turbina (Cor. tesÍa de Brown Boveri Company).

Principio de operación 3 11

en el rotor un conjunto adicional de devanados que semejan la jaula de un motor de inducción. A este conjunto se le llama devanado amortiguador y se ilustra en las figuras 7.1,7.4 y 7.6. Cuando la velocidad del rotor difiere de la síncrona, se inducen \,;orrientes en el devanado de amortiguamiento, que actúa como el rotor de jaula del motor de inducción, produciendo un par que restaura la velocidad síncrona. Además,las barras de amortiguamiento proporcionan un medio de arrancar la máquina como motor síncrono, que de otra manera no sería de arranque propio. En resumen, las figuras 7.1 a 7.9 muestran los diversos aspectos estructurales de diferentes tipos de máquinas síncronas. En principio, la máquina tiene tres devanados eléctricos: la armadura, el campo y el amortiguador, localizados en el estrator yen el rotor, como se ilustra en la figura 7.1. En la sección que sigue se inicia el estudio de una máquina síncrona en su forma más simple. 7.2 PRINCIPIO DE OPERACION Se mencionó antes que una máquina síncrona es capaz de operar como motor generador. En esta sección se considerarán los aspectos cualitativos de la acción de una máquina síncrona,.primeramente como motor yen seguida como generador. 7.2.1 Operación de un motor síncrono De la discusión de la sección 7.1, se observa que hay semejanza entre el rotor de polo saliente de una máquina síncrona y el de Uno de reluctancia, discutido en el capítulo 4. De esta manera, de los principios de almacenamiento y conversión de energía desarrollados en el capítulo 4, puede mostrarse cómo una máquina síncrona polifásica opera en calidad de motor. Se demostró en el último capítulo que una excitación trifásica, como la que se encuentra en el estator de un motor de inducción, da lugar a un campo magnético rotatorio en el entrehierro de aire de la máquina. Con referencia a la figura 7.l0a, se tendrá un campo magnético rotatorio en el entrehierro de aire de una máquina de polo saliente cuando sus devanados de estator (o de armadura) se alimentan de una fuente trifásica. El rotor tendrá entonces siempre una tendencia a alinearse con el campo, con el objeto de ofrecer la trayectoria de menor reluctancia. Así, si el campo gira, el rotor tenderá a girar con él. En la figura 7.10b se ve que un rotor redondo no tenderá a seguir al campo magnético rotatorio, pues el entrehierro de aire uniforme ofrece la misma reluctancia en todo su alrededor y el rotor no tiene ninguna dirección de preferencia para alinearse con el campo magnético. Este par, que se tiene en la figura 7.10a pero no en la 7.l0b, se de-

312 Máquinas síncronas Eje rotor

Eje de la fase a

+----t---

(a)

(b)

FIGURA 7.10 a) Un motor síncrono trifásico con rotor de polos salientes. b) Un motor síncrono trifásico con rotor redondo.

nomina par de reluctancia. Está presente en virtud de la variación de la reluctancia alrededor de la periferia de la máquina. Considérese en seguida que el devanado de campo (figura 7.10a o b) se alimenta con una fuente de c.c. que produce un campo magnético de rotor con polaridades definidas. Por el principio de alineamiento de campos (capítulo 4), se concluye que cuando se excita el rotor, tiende a alinearse con el campo de estator y tenderá a girar con el campo magnético rotatorio. Se observa que para un rotor excitado, tanto un rotor saliente

Principio de operación 3 13

como uno redondo tenderán a girar con el campo magnético rotatorio, aunque el primero tendrá un par de reluctancia adicional en virtud de la protuberancia. En una sección posterior, se establecerán expresiones para el par electromagnético en una máquina síncrona, atribuible a la excitación de campo y a la protuberancia. Hasta este punto se ha indicado el mecanismo para producir par en una máquina de rotor redondo y en una de protuberancia. Para recapitular, puede decirse que el campo magnético rotatorio de estator tiene una tendencia a "arrastrar" consigo al rotor, como si un polo norte en el estator "se ligara" con uno sur en el rotor. Empero, si el rotor está en reposo, los polos de estator tenderán a hacer que el rotor gire en una dirección y en seguida en la otra, al girar ellos y barrer a través de los polos del rotor. Por consiguiente, un motor síncrono no arranca por sí mismo. En la práctica, como se mencionó antes, el rotor lleva barras amortiguadoras que actúan como la jaula en un motor de inducción, proporcionando con ello un par de arranque. Una vez que arranca el rotor, alcanzando casi la velocidad síncrona, se orienta en posición con los polos de estator. El rotor se empareja con el campo magnético rotatorio y gira con la velocidad síncrona, dejando de operar las barras de amortiguamiento. Cualquier diferencia con la velocidad síncrona se traduce en corrientes inducidas en las barras de amortiguamiento, que tienden a restaurar la velocidad síncrona. Las máquinas que carecen de estas barras o máquinas muy grandes que sí las tienen, pueden arrancarse mediante un motor auxiliar. Se discutirán posteriormente las características de un motor síncrono, aunque primeramente se analizará la operación de una máquina síncrona accionando como generador. 7.2.2 Operación de un generador síncrono

Como el generador de c.c., un generador síncrono funciona en base de la ley de Faraday. Si el flujo que eslabona a la bobina cambia en el tiempo, se induce en ella un voltaje. Dicho de otra manera, un voltaje se induce en un conductor si corta líneas de flujo magnético. Considerando la máquina que se muestra en la figura 7.l0a sin tomar en cuenta los efectos de distribuciones de devanado, (es decir, se consideran bobinas concentradas para cada fase), se encuentra que la distribución de densidad de flujo en el entrehierro de aire, producida por el devanado de campo, es

(7.2) El flujo que eslabona una bobina de N vueltas, de radio r y longitud axial se calcula como sigue: eslabonamiento de flujo por polo = A= N

f

'TT/2

-'TT/2

B( O )/rdO

(7.3)

314 Máquinas slncronas Las (7.2) Y (7.3) dan

(7.4) De la figura 7.10a es obvio que cuando a = 0, A. es máxima y nula para a = O. El valor de A. para cualquier posición a de rotor es, de (7.4),

(7.5) El voltaje inducido en la bobina de N vueltas, correspondiente a la fase a, en nuestro caso se obtiene de la ley de Faraday y de acuerdo con (7.5), da v = - dA = _ dA da =2NB /rwsena a dI da dI m

donde da/dt wt y 2Bm Ir

(7.6)

= w, es la velocidad del rotor. Además, puede ponerse a = = ifl en (7.6) para obtener la forma final para el voltaje, como (7.7)

donde V m = W N ifl. La conclusión evidente es que para una distribución de densidad de flujo sinusoidal producida por el devanado de campo en el rotor, el voltaje inducido en la fase "a" es sinusoidal y su frecuencia! = 21T /w depende de la velocidad de rotor. Se considerará en seguida la máquina de rotor redondo de la figura 7.10, donde la distribución de densidad de flujo producido por la fmm de rotor en el entrehierro de aire es uniforme. De acuerdo con la regla del "corte de flujo", el voltaje inducido en la bobina de N vueltas es, para un instante dado, Va

= B/U i.

(7.8)

donde U 1, B Y I son mutuamente perpendiculares. La velocidad lineal U se relaciona con la angular mediante U = rw y, según la figura 7.1 Ob, su componente vertical es U 1 = U sen a. Sustituyendo estas expresiones en (7.7) con a = wt, se obtiene

(7.9) lo que implica también una generación de voltaje sinusoidal en la fase a. 0 Puesto que las fases b y e están desplazadas de la a en 120 , pueden escribirse los voltajes correspondientes como

(7.10)

Algunas consideraciones prácticas 315

(7.11) En la figura 7.11 se representan gráficamente las expresiones anteriores.

7.3 ALGUNAS CONSIDERACIONES PRACTICAS

Antes de considerar los métodos de análisis usuados en máquinas síncronas que llevan a la determinación de sus características operacionales, se revisarán algunas de las suposiciones hechas o involucradas en las discusiones anteriores. En primer lugar, al obtener la ecuación de voltaje en la sección 7.2.2 se representó un devanado de fase por una bobina de N vueltas. En realidad, no es esto válido, como se analizó en el capítulo 6. Se insiste, de la sección 6.1, que un devanado de fase está distribuido alrededor de la periferia del estator. El paso de bobina del devanado es una fracción del paso polar, lo que significa que el devanado puede tener un paso fraccionario. Para tener en cuenta la distribución de devanado y el paso fraccionario, se utiliza el factor de devanado Kw < 1, dado por

C·12) mediante el cual se reduce el voltaje en una bobina de paso completo y concentrada de N vueltas. En (7.12), Kd es el factor de distribución y Kp es el de paso, definidos en la sección 6.1. Al distribuir un devanado de fase en las ranuras alrededor del estator, la distribución resultante de fmm se diseña teóricamente para aproximarse a una sinusoide tanto como sea posible. No obstante, en la realidad un devanado de armadura contiene armónicas, como indica la representación en serie de Fourier de la distribución de fmm. Para mayores detalles, véanse las referencias 2 y 3. En segundo lugar, para una máquina con polo saliente, se supuso una distribución de flujo sinusoidal en el entrehierro de aire. Esta es una suposición ideal nuevamente; en la práctica, la distribución de campo es no sinusoidal y se aproxima a la forma mostrada en la figura 7.12b. Sin considerar la reluctancia de la porción de hierro del circuito magnético, se encuentra que la permeancia de cada tubo de flujo es proporcional a la razón entre su sección transversal media y su longitud media. Así, la distribución de permeancia se conoce alrededor de la periferia del estator. Conocida la distribución de fmm, puede obtenerse la distribución de densidad de flujo. En tercero, hasta este punto de dis.cusiones no se ha hecho ninguna consideración acerca de la presencia de ranuras y dientes. De las figuras 7.13 a y b, es evidente que hay una variación de permeancia para el flujo de entrehierro de aire a causa de las ranuras y dientes, que dará lugar a que la forma de campo contenga fluctuaciones o rizos, como se muestra

Voltaje inducido

FIG URA 7.11 Voltaje trifásico producido por un generador síncrono trifásico. Estator

o Rotor

\

\--\

(a)

Posición a lo largo del estator (b)

FIG URA 7.12 a) Distribución espacial de campo: líneas de flujo; líneas equipotenciales. b) Distribución de densidad de flujo.

316

Algunas consideraciones prácticas 3 17

Polo de campo

ranuras del

t _ _ Corriente de campo

FIGURA 7.14 Características en circuito abierto de una máquina síncroná.

en la figura 7.13c, llamados rizos de diente. Estos rizos crean armónicas en las fems inducidas, de frecuencias superiores a los 60 Hz, (por ejemplo de 875 Hz). Las frecuencias, a su vez, originan ruido magnético. Un diseño adecuado, como disponer de un número grande de ranuras con aberturas pequeñas comparadas con el entrehierro de aire, o de ranuras oblicuas, o bien, con un número de ranuras múltiplo no entero del número de polos, reduce los rizos de diente en un máquina síncrona. En la figura 7.14 se muestra, finalmente, el efecto de saturación sobre el voltaje en circuito abierto. 7.4 CARACTERISTICAS DE OPERACION DE MAQUINASSINCRONAS

Se examinarán ahora algunas de las características de operación en régimen estacionario de las máquinas síncronas desde un punto de vista cuantitativo. Se considerará, por convenir así, la operación generador independientemente de la motor. Los detalles del método de análisis para la máquina con rotor redondo diferirán en cierta manera del procedimiento para una con polo saliente. En cualquier caso, el análisis necesita de los parámetros de máquina, que deben identificarse antes de proceder con los pormenores analíticos. Puesto que por ahora sólo se analiza el comportamiento en estado estacionario de la máquina, no será necesario tomar en cuenta las constantes de circuito del campo y de los devanados amortiguadores. La presencia del devanado de campo se denotará con el flujo producido por la excitación de campo. Respecto al devanado de armadura, se le representará sobre una

Caracterlsticas de operación de máquinas slncronas 319 Conexión terminal

Flujo de dispersión en la conexión terminal

Conductor activo

Flujos de dispersión en ranuras

FIGURA 7.15 a) Trayectoria de flujo de fuga en la conexión terminal.b) Trayectorias de flujo de fuga en ranuras.

base por fase (como se hizo con el motor de inducción en el capítulo 6). Desde luego que este devanado posee resistencia, aunque el valor óhmico de ella debe incluir los efectos de la temperatura de operación y de las corrientes alternas que fluyen en los conductores de armadura (que dan lugar, por ejemplo, al efecto pelicular), por lo que el valor de la resistencia de armadura se hace mayor en comparación a su resistencia en c.c. A este valor mayor se le conoce como resistencia efectiva y se le denota con ra' Como valor aproximado de esta ra se toma 1.6 veces el valor de la resistencia c.c. Se consideran en seguida las reactancias asociadas al devanado de armadura. En primer término, a la de fuga la originan los flujos de fuga que eslabonan sólo a los conductores de armadura, en virtud de las corrientes en ellos. Estos flujos no eslabonan con el devanado de campo. Como en el motor de inducción, se divide, por conveniencia, esa reactancia de fuga en: a) reactancia de fuga por conexión terminal; b) reactancia de fuga por ranura; c) reactancia de fuga por zig-zag de punta de diente y d) reactancia de fuga por cinturón. No todas estas componentes tienen importancia en cualquier máquina síncrona; en la mayoría de las grandes máquinas las dos últimas representan una parte pequeña de la reactancia de fuga total.

320 Máquinas sincronas

En las figuras 7.15 a y b se muestran las trayectorias de flujo que contribuyen a las reactancias por conexión terminal y por ranura. Con Xa se denota la reactancia de fuga total del devanado de armadura por fase. Para proceder con el análisis, considérese en primer término un generador sÍncrono con rotor redondo. Se introducirá además el concepto de rectancia síncrona, el parámetro de mayor importancia para determinar las características en régimen estacionario de una máquina síncrona. 7.4.1 Operación de un generador síncrono de rotor redondo

Para iniciar, se desea puntualizar que se estudiará la máquina sobre una base por fase, implicando operación balanceada. De este modo, considérese una máquina con rotor redondo operando como generador sin carga. Sea V o el voltaje de fase en circuito abierto para una cierta corriente Ir de campo. Aquí, V o es el voltaje interno del generador. Se supone que If es de tal magnitud que la máquina opera bajo condición nO saturada. En seguida, cortocircuítese la armadura en las terminales, conservando sin modificar el valor If de la corrriente de campo y mídase la corriente de fase la de armadura. En este caso, el voltaje interno total V o cae a través de la impedancia interna de la máquina. Matemáticamente (7.13)

y Zs es la llamada impedancia síncrona. Una parte de Zs es 'a y la otra la reactancia X s ' conocida como reactancia síncrona; es decir, Zs= ra + jX,

(7.14)

En (7.14) X s es mayor que la reactancia de fuga de armadura analizada con anterioridad. ¿De dónde proviene la reactancia adicional? En la dis" . cusión que sigue se intentará responder esta pregunta. Supóngase que el generador suministre una corriente de fase la a una carga con factor de potencia unidad y con un voltaje terminal V t voltjfase. Esta relación se muestra en el diagrama fasoríal de la figura 7.16 donde Va es la suma fasorial de V t y la caída debida a la resistencia de armadura y a la reactancia de fuga de armadura. Nótese que se tienen ahora dos fmms: Fa' atribuible a la corriente de armadura y Ff a la corriente de campo presente en la máquina. Para encontrar la fmm Fr que produce el voltaje Va' véase en la figura 7.17 la característica en circuito abierto del generador, que muestra la Fr correspondiente a Va' El flujo producido por una fmm está en fase con ella. De acuerdo con la ley e = -N dcjJjdt, el voltaje producido por un cierto flujo está, no obstante, atrasado en 90° respecto a la fmm. Por consiguiente, se traza Fr 90° adelante de Va y Fa en fase con la' como se ilustra en la figura 7.16. La fmm Fn se conoce como fmm de ,eacción

Características de operación de máquinas síncronas 321

de armadura. El campo debe suministrar una fmm suficiente para sobrepasar a Fa' de tal suerte que se disponga de una F r neta para producir a Va' A causa de Fa' se traza a partir de F r , una componente igual y opuesta Fa de la fmm de campo, como se muestra en la figura 7.16. El fasor Ff es entonces la fmm de campo total en la máquina. Correspondiente a esta fmm, el voltaje en circuito abierto del generador es V o ' dado por la figura 7.17. Este voltaje se conoce como fem inducida nominal y se muestra también en la figura 7.16. De la geometría del diagrama de fasores, puede verse con facilidad que los triángulos OST y OQR son semejantes. Se observa también que QR es perpendicular a OP y que debe pasar por P, pues OP = V t + la r a • De esta manera, QRP es una línea continua y se tendrá (7.15) donde X s ' reactancia síncrona, concuerda con las definiciones (7.12) Y (7.13). La reactancia adicional, aunada a la Xa (figura 7.16), se introduce por la reacción de armadura. Por lo tanto, la reactancia síncrona es la suma de la reactancia de fuga de armadura y la reactancia de reacción de armadura. En una máquina síncrona real, excepto en las sumamente pequeñas, se tiene siempre X. » ra X s « ra , en cuyo caso Zs === jX•. Esta restricción se usará en la mayor parte del análisis. Entre las características de régimen estacionario de un generador síncrono, las más importantes son las de regulación de voltaje y de ángulo de potencia. Como se hizo para un transformador y un generador de c.c., se define aquí la regulación de voltaje de un generador síncrono para una carga dada, como

v-vIX 100

regulación de voltaje en % = o

VI

(7.16)

FIGURA 7.16 Diagrama fasorial para un generador con rotor redondo con factor de potencia unidad (para definir X.)

322 Móquinas síncronos

o

V. 1 - - - - - - 6 1 '

3! u

::l "O

oS

.,

¡ o

>

t ~Fmm

FIGURA 7.17 Características en circuito abierto de un generador síncrono.

donde V t es el voltaje terminal con carga y V o es el terminal sin ella. Obviamente, para un V t dado, puede encontrarse V o de (7.14) y de aquí la regulación de voltaje, como se ilustra en el ejemplo que sigue. Ejemplo 7.1

Calcular la regulación de voltaje en % para un turboalternador trifásico, conectado en Y de 2500 kV A, 6600 V, operando en carga total y con un factor de potencia de 0.8 en atraso. La reactancia síncrona por fase y la resistencia de armadura son lOA y 0.0071 Ohms, respectivamente. Es obvio que se tiene X s »ra' En la figura 7.18a se muestra el diagrama fasorial para el factor de potencia en atraso, no considerando el efecto de ra' Los valores numéricos son las siguientes ~=

6600 - - =3810 volt

V3

1 = 2500 x 1000 =218.7 A a V3 x6600

De (7.14), Vo=381O+ 218.7(0.8 - jO.6)jlO.4= 5485 L 19.3 0 y

., d e vo1t' regu1aClon aje en % = 5485-3810 3810 x 100 = 4407 10

Características de operación de máquinas síncronas 323 Ejemplo 7.2

Repetir los cálculos anteriores con el mismo factor de potencia, pero en adelanto. En este caso, se tiene el diagrama de fasores de la figura 7.18b, del que se obtiene Vo= 3810+ 218.7(0.8 + jO.6)jlO.4 = 3048 L 36.6° y

., d e volt' regu1aClOn aje en % = 3048-3810 3810

X

100 = - 2001 70

Se observa de los dos últimos ejemplos que la regulación de voltaje depende del factor de potencia en la carga. A diferencia de lo que acontece en un generador de c.c., la regulación de voltaje para un generador síncrono puede áun llegar a ser negativa. El ángulo entre V o y V t se define como 8, ángulo de potencia. Para justificar esta definición, reconsideremos la figura 7 .18b, de la cual se tiene (7.17) De (7.16) Y (7.17), (7.18) que muestra que la potencia interna de la máquina es proporcional a sen &. De la ecuación (7.18) se dice a menudo que representa la característica de ángulo de potencia de una máquina síncrona.

I

a "'......

...... "'"\

il.X.~X. ~:::""....--I.._-----"-¡I Vt

.........

.......... -J

/

I

I

/

V

t

I (h)

(a)

FIGURA 7.18 Diagramas fasoriales. a) Factor de poetencia en atraso. b) Factor de potencia en adelanto.

324 Máquinas s(ncronas

7.4.2 Operación de un motor síncrono con rotor redondo

Salvo para ciertos cálculos de precisión, puede despreciarse la resistencia de armadura en comparación con la reactancia síncrona. Es por esta razón que el circuito equivalente por fase en régimen estacionario de una máquina síncrona se reduce al que se muestra en la figura 7.19a. Nótese que este circuito es similar al de una máquina de c.c., donde se ha sustituido la resistencia de armadura en c.c. por la reactancia síncrona. En la figura 7.19a se indica el voltaje terminal V t , el voltaje V o de excitación interna y la corriente la de armadura "penetrando" a la máquina o "emergiendo" de ella, dependiendo del modo de operación: "penetrando" para motor y "emergiendo" para generador. Con la ayuda de este circuito y (7.18), se estudiarán algunas de las características de operación en régimen estacionario para un motor síncrono. En la 7 .19b se muestran las características de ángulo de potencia, dadas por (7.18). Aquí, la potencia y la o positivas implican la operación generador, mientras que una (j negativa corresponde a la operación motor. Como o es el ángulo entre V o y V t , V o adelante a V t en un generador, mientras que en un motor V t adelante a V o ' La ecuación de balance en voltaje en un motor es (7.19)

Si el motor opera en potencia constante, (7.17) y (7.18) requieren que (7.20)

Se insiste en que V o depende de la corriente de campo Ir' Considérense dos casos: 1) Ir se ajusta de tal suerte que V o < V t y la máquina está subexcitada; 2) Ir se incrementa hasta un valor para el cual V o > V t y la máquina llega a sobreexcitarse. En la figura 7.20a se muestran las relaciones

a-

1

Potencia

Motor

x,

------~~~-------+ +

---¡;;-

Generador

v,

(a)

-

- - - Generador

M otor (h)

FIGURA 7.19 a) Un circuito equivalente aproximado. b) Características de ángulo de potencia de una máquina síncrona.

Caracteristicas de operación de máquinas síncronas 325

""

" /" \ \

\

\

\

\

\

\

,

v,\

(a)

(b)

FIGURA 7.20 a) Diagramafasorial para la operación motor. (V'o. ¡'a. ep' yo') corresponden a operación subexcitada. (V" o. ¡"a. ep" y o") corresponden a operación sobreexcitada. b) Curvas V de un motor síncrono.

voltaje-corriente para los dos casos. Para V o > V t en potencia constante, o es mayor que la o para V o < V p de acuerdo con (7.20). Obsérvese que un motor subexcitado opera en un factor de potencia en atraso (la está atrasada respecto a V t ), mientras que uno sobreexcitado opera con un factor de potencia en adelanto. En ambos casos, el voltaje terminal y la carga en el motor son los mismos. De este modo, se observa que el factor de potencia operante del motor se controla variando la excitación de campo y, consecuentemente, alterando a V o ' Esta es una propiedad sumamente importante de los motores síncronos. En la figura 7.20a se muestra también

326 Máquinas síncronas

el lugar de la corriente de armadura en carga constante, de acuerdo con (7.20). De aquí pueden obtenerse las variaciones de corriente la de armadura con la de campo 1f (correspondiente a V o ) y esto puede hacerse para diferentes cargas, como se ilustra en la figura 7.20b. Estas curvas se conocen como las curvas V del motor síncrono. Una de las aplicaciones de un motor síncrono es la corrección del factor de potencia, como se muestra en ejemplo siguiente. Ejemplo 7.3

Una carga trifásica, conectada en Y, absorbe una corriente de 50 A en un factor de potencia de 0.707 en atraso, con 220 V entre líneas. Un motor síncrono, trifásico, conectado en Y con rotor redondo y con reactancia síncrona de 1.27 Ohms por fase, se conecta en paralelo con la carga. La potencia desarrollada por el motor es de 33 kW en un ángulo de potencia de 30° . Sin considerar la resistencia de armadura, calcular: a) los kVA reactivos del motor y b) el factor de potencia global del motor y la carga. En la figura 7.21 se muestran el circuito y el diagrama fasorial, sobre base por fase. De (7.18), se tiene

+ Motor

Vt

(a)

,,/\

", 1

\ \.

\ \

,

\

v,\

FIGURA 7.21 a) Diagrama de circuito. b) Diagrama de fasores.

Características de operación de máquinas síncronas 327 1

Pd ="3

220

Vo

x 33,000= V3 1.278 sm30

o

lo que da V o = 220 V. Del diagrama fasorial, laXs = 127 o la = 127/1.27 = 100 A Y l/>a = 30°. Los kVA reactivos del motor =ftVla =

=..j3

X 220 X 100 sen 30° 1000

= 19 kVAr.

El ángulo l/> de factor de potencia global lo da

de donde l/> = 7° Y cos l/> = 0.992 en adelanto. 7.4.3 Máquinas síncronas de polos salientes

En lo anterior, se analizó la máquina con rotor redondo y se utilizó ampliamente el parámetro de máquina que se definió como reactancia síncrona. En virtud de la protuberancia, la reactancia medida en las terminales de una máquina con rotor saliente variará en función de la posición de roto~, lo que no sucede en la de rotor redondo (ver también el capítulo 4: Motor de reluctancia). Para superar esta dificultad, usamos la teoría de dos reacciones, propuesta por André Blendel (ver también la sección 9.5). La teoría propone descomponer las fmms de armadura dadas en dos componentes mutuamente perpendiculares, una a lo largo del eje del polo saliente de rotor, conocido como eje directo (o eje d) Y la otra en cuadratura, conocida como eje de cuadratura (o eje q). La componente de fmm F d' según el eje d, puede ser magnetizante o desmagnetizante y la F q , a lo largo del eje q, se traduce en un efecto magnetizante cruzado. Así, si la amplitud de la fmm de armadura es Fa' entonces (7.21) y

(7.22)

donde 1/1 es el ángulo de fase entre la corriente la de armadura y el voltaje interno V o (o de excitación). En términos de distribución espacial, se muestra en la figura 7.22 las fmms y 1/1. Los efectos de Fd YF q se manifiestan en la producción de voltajes. En vía de ilustración, considérese un generador de polo saliente, con un voltaje terminal alimentando a una carga

328 Máquinas slncronas

Eje - d

Eje-q

FIGURA 7.22 Fmm de armadura y sus componentes d y q.

con factor de potencia en atraso (cos cp ) y absorbiendo una corriente de fase la' De acuerdo con la condición de operación (corriente de campo dada), se conoce también el voltaje V o sin carga a partir. de las características correspondientes. Estas características (Vt • VOl la y (J ) se muestran en la figura 7.23. Para construir este diagrama, se toma a V o como fasor de referencia y se desprecia la resistencia de armadura. Ahora, con carga en virtud de la sola reacción de armadura, V o se reducirá a Va' en función de Vd y V q , originados por Fd y F q , respectivamente. De Va se obtiene V t restando la caída de reactancia de fuga en armadura laXa' Si no se toma en cuenta a Va' puede afirmarse que la diferencia entre V o y V t es atribuible a la reactancia de reacción de armadura y a la fuga de armadura; es decir, a la reactancia síncrona. Puede resolverse la en sus componentes a lo largo de los ejes d y q: Id e Iq , respectivamente, del mismo modo que la caída IaXs mostrada en la figura 7.23 como Id X d e Iq X q . Se completa así el diagrama fasorial (ver el ejemplo siguiente). Pueden darse interpretaciones físicas a las reactancias X d y Xq ; a saber, las reactancias de eje directo y de eje de cuadratura y los valores máximo y mínimo de la reactancia síncrona de una máquina de polos salientes respectivamente. Estas reactancias pueden medirse experimentalmente como se verá en una sección posterior. Las discusiones precedentes son válidas sobre una base por fase para una máquina balanceada. Con el ejemplo que sigue se mostrarán los detalles de algunos de los cálculos.

Ejemplo 7.4

Un generador síncrono de polos salientes trifásico, conectado en Y, de 20 kVA, 220 V Y 60 Hz, alimenta una carga nominal en un factor de potencia de 0.707 en atraso. Las constantes por fase de la máquina son ra = 0.05 n y X d = 2 X g = 4.0 n. Calcular la regulación de voltaje para la carga especificada.

Caracteristicas de operación de máquinas sincronas 329

FIGURA 7.23 Diagrama fasorial de una máquina de polos salieI)tes

V¡ = 220 = 127 volt

v'3

1 = a

20,000

v'3 x220

= 52.5 A.

=cos-10.707=45° De la figura 7.23, se tiene

Id = Iasen( S + cp) Iq=Iacos(S+cp) V/senS = IqXq = IaXq cos( S + cp) o bien

52.5x2xO.707 =037 127 + 52.5 X 2 X 0.707 . o sea

S=20.6° I d = 52.5sen(20.6 +45) =47.5 A.

IdXq =47.5 x4= 190.0 V Vo = V/cosS+IdXd = 127cos20.6+ 190=308

330 Máquinas síncro1llls

La regulación de voltaje en % =

v.-v. oV. / X 100% /

El ejemplo anterior muestra la manera en que el diagrama de faso res de la figura 7.23 puede emplearse para determinar la regulación de voltaje de un generador síncrono de polo saliente. De hecho, el diagrama fasorial describe las características de operación completas de la máquina. Por ejemplo, para obtener las características del ángulo de potencia de una máquina de polo saliente, operando como generador o como motor, véase la figura 7.23. De esta figura, se tiene, despreciando a ra y las pérdidas internas, potencia de salida = V,Iacos«/>=potencia desarrollada =Pd

(7.23)

y

IqXq= V,sen8

(7.24)

IdXd = Vo - V¡ cos l)

Además, I d =l,pen( 8+«/»

(7.25)

Iq = la cos( 8 -«/»

Sustituyendo (7.25) en (7.24) y resolviendo para la cos = ysen8+ 2X sen28- 2X sen28 d

q

(7.26)

d

Finalmente, la sustitución de (7.26) y (7.23) da

(7.27)

Esta variación de la potencia desarrollada Pd , en función del ángulo de potencia 5 se muestra en la figura 7.24. Obsérvese que la potencia resultante se forma con la potencia debida a la protuberancia -el segundo término en (7.27) y la que origina la excitación de campo- el primer término en :(7.27). Obviamente, cuando X d = X g , la máquina carece de protuberancia y sólo el primer término en (7.27) es diferente de cero, lo que representa

Transitorios en máquinas síncronas 331

Potencia debida a la saliente,

V? -2 (....!.... - ...1..) sen 28 Xq X d

\ / '\ \

\

).,

Potencia debida a la excltacl6n de campo. VtVo sen 8 Xd \

/ \ ----~~--~~--~~--~----~-----8

Motor

-

Generador

FIGURA 7.24 Características de ángulo de potencia de una máquina de polos salientes.

la característica de ángulo de potencia en una máquina con rotor redondo. Por otra parte, si no hubiera excitación de campo, lo que conduciría a Vo = 0, se anularía el primer término de (7.27). Se tendrían entonces las características de ángulo de potencia de una máquina de reluctancia, dadas por el segundo término, descutidas también en el capítulo 4. Como en una máquina con rotor redondo discutida con antelación, las características de ángulo de potencia dadas por (7.27) corresponden tanto a la operación motor como a la generador. El término & es positivo para el último y negativo para el primero.

7.5 TRANSITORIOS EN MAQUINAS SINCRONAS

En las secciones precedentes se concentró la atención sobre el comportamiento en régimen estacionario de las máquinas síncronas. En esta sección se verán sucintamente algunos casos que involucran transitorios en máquinas síncronas. Son de particular interés a) el cortocirucito repentino en las terminales de armadura de un generador síncrono y b) los transitorios mecánicos originados por un cambio súbito en la carga de la máquina. Hay otros casos numerosos que presentan transitorios en máquinas síncronas, pero que no se considerarán aquí. Se sabe, de consideraciones anteriores, que puede determinarse la operación de una máquina si se conocen sus parámetros para una determinada condición. Por ejemplo, se han expresado ya las características de ángulo

332 Máquinas síncronas

de potencia en régimen estacionario de una máquina síncrona de polos salientes en términos de las reactancias de ejes d y q. En forma semejante, las constantes mediante las cuales se conoce el comportamiento transitorio de una máquina síncrona, son las reactancias transitorias y subtransitorias y las constantes de tiempo pertinentes. Estas cantidades se definen en la subsección que sigue, refiriéndolas al estudio de un cortorcircuito de armadura. 7.5.1 Cortocircuito repentino en las terminales de armadura

Al principio no se supo so saturación y se despreciaron las resistencias de todos los devanados: de armadura, de campo y de amortiguamiento. De esta manera, sólo permanecieron las inductancias, implicando que el flujo que eslabonaba con un circuito cerrado (o armadura) no variara instantáneamente, de acuerdo con el teorema de eslabonamiento de flujo constante. Dicho de otro modo, la suma de los eslabonamientos de flujo era constante para cada devanado. Con estas suposiciones en mente, considérese una máquina de rotor redondo (figura 7.25) Y obsérvese la fase a y el devanado de campo. Sea If la corriente de campo para t = O. Con anterioridad a t = O, se supone abierta laa armadura. En t = O, el devanado de armadura se cortocircuita súbitamente, cuando el eje de fmm de la fase a se encuentra en ángulo recto con el del devanado de campo. Esto tiene lugar de tal suerte que no nay acoplamiento mutuo entre los devanados. Es obvio que en t = O el flujo Aa que eslabona con la armadura es cero. Empero, el que eslabona con el devanado de campo Af = L/f , donde Lf es la inductancia de devanado de campo. Se puede separar a Lf de tal manera que (7.28)

donde L¡ es la inductancia de fuga de campo y Lad es la mutua entre los devanados de campo y de armadura. Así, Lad corresponde también a la reactancia de reacción de armadura. Puede reescribirse d f usando (7.28) como (7.29)

donde

7f

=LdLad

es el coeficiente de fuga de campo.

Transcurrido un tiempo t, supóngase que el rotor gira un ángulo e (ver figura 7.25) en cuyo caso ia e ir + Ir fluirán a través de 1.os devanados de campo y armadura, para mantener los eslabonamientos de flujo con estos devanados. Por lo tanto, para el devanado de armadura se tiene (7.30)

~ • I \

Transitorios en máquinas sincronas 333

IEje del campo I

Eje de la fase a

FIGURA 7.25 Máquina trifásica de rotor redondo (sólo la fase a y el devanado de campo llevan corrientes).

donde 7 a = X¡/wL ad es el coeficiente de fuga de armadura. De igual m mera, para el devanado de campo, se tiene

A¡= (~+ /¡ )LaA 1 + 7¡) + iaLadsenO =/¡La

Al + 7¡)

(7.31 )

Al resolver (7.30) y (7.31) para ia e ir se obtiene

.

[ (1 + 7¡ )senO] /¡

1= a sen

20-(1+7 )(1+7¡) a

.

(sert- O) /¡

I¡= - sen20- (1 + 7 0 )(1 + 7¡) Los valores máximos de estas corrientes tienen lugar en caso, se tiene

(7.32)

(7.33)

e = 1rJ2. En este (7.34)

(i)

=

f máx

/ ¡

7o + (1 - 7o )7¡

(7.35)

...

334 Máquinas slncronas

Al multiplicar el numerador de (7.34) por wL ad se obtiene

(7.36) donde V o = W La d 1f es el voltaje inducido (interno), X f = W La d r f es la reactancia de fuga del campo y XI = wLad -ra es la reactancia de fuga de armadura. El circuito que corresponde a (7.36) se muestra en la figura 7.26 Y la reactancia de entrada de este circuito es la reactancia transitoria X'd de eje directo. En seguida, para incluir los efectos de los devanados de amortiguamiento en el eje directo, observamos que el efe=6.786 x 1O- 6 (NI)fwebers, (NI)f=4.2 x 10- 3 +4.1 x 1O- 1 (NI)fwebers, (NI)f > 700 483

(5.36)

484 Apéndice 111 Caracter{sticas magnéticas

/

5

/

__ ---- --r/

__ -

--

/

4

/

E

É ~

o 3 c.

oc.

~ 2 o ~

LL

oL-__

~

____

400

~

____L -__

800

1200

~

____- L____L -__

1600

2000

2400

~

____- L_____

2800

3200

Ampere-vueltas por polo del campo

FIGURA III. 1. Curva del flujo contra ampere-vueltas y aproximación con líneas rectas.

Podría haberse introducido una tercera sección para describir con más exactitud a la curva en la región de la rodilla de 1> contra la característica NI. e) Existen varios métodos para representar una curva, como la figura IU.l, por medio de series de potencias de NI (o de H). En general, se requiere un mínimo de tres términos para una representación de este tipo. En muchos sistemas de computadora que pueden ser accesibles al lector, los programas de soporte lógico para análisis de regresión pueden dar las expresiones de serie de potencias para características específicas de B contra H. d) Se han propuesto muchas expresiones analíticas para la característica de B contra H, una sumamente útil y que se ha usado en métodos de elementos finitos para el análisis de circuitos magnéticos es 1

donde K 1, K 2 Y K3 se determinan haciendo que la ecuación (5.38) pase por tres puntos experimentales en una característica B-H de laboratorio. e) Elementos finitos. Hay muchos programas de computadora para el análisis general de circuitos magnéticos usados en dispositivos electromecánicos. El uso principal de la técnica de elementos finitos es el establecimiento y trazo de gráficas de líneas equipotenciales

Apéndice 11J 485

y de flujo, el cálculo de energía e inductancia y de fuerza. Un programa disponible en muchos sistemas de computadora para el usuario es el AOSjMAGNETIC0 2 • B. TRANSFORMADORES E INDUCTORES

Hay una serie de programas de soporte lógico para el diseño de transformadores e inductores disponible en muchos sistemas de computadora para el usuario: 1. TRANS, programa general de diseño de transformadores. 2. INDUCTOR, programa de diseño de inductores. 3. TOPT, técnica de optimización de transformadores. 4. FERRO, programa de diseño para transformadores ferro-resonantes.

Bibliografía 1. J. R. Brauer, "Simple Equations for the Magnetization and Reluetivity Curves of Steel," IEEE Transactions on Magnetics, Nueva York, Enero 1973. 2. A. O. Smith, Broehure, "Engineering Consulting Serviee,"MKT-l10577, Mayo 1977. 3. Available from Optimized Program Serviees, Ine., Berea, OH.

Indice

Coeficiente de acoplamiento, 64 Condiciones de frontera, 60 Conmutador, 20, 181, 208, 213 Control de estado sólido, 347 activación luminosa, 366 apagado de compuerta de thyristor, 366 cicloconversor,426 circuitos, recortadores, 411 conmutación paralelo de thyristor, 380 conmutación por thyristor, 373 conmutación serie de thyristor, 375 conmutador controlado por compuerta, 366 conmutador controlado por silicio, 390 controlador de eslabonamiento c.c., 418 control de motor de c.a., 416 control de razón de tiempo, 410 Darlington de potencia, 371 diodo de avalancha, 371 diodo de volante libre, 360, 399 diodos Zener, 371 disparo de compuerta de thyristor, 389 encendido de compuerta de thyristor, 385,387 formas de onda, 353 impulso de base de thyristor, 390 inversor de onda cuadrada, 418 inversores, 420, 425 lámina de datos de thyristor, 365, 366 lámina de datos de transistor de potencia, 369 ondas recortadas, 354 protección de semiconductor de potencia, 391 recortador, 375,409

A

Aceite, aislamiento, 85 Acoplamiento de impedancias, 107 Aislador magnético, 46, 51 Aislamiento, 19 Alambre magneto, 87 Analogía fuerza-corriente, 169 Angulo eléctrico, 195, 201 Angulo del par de torsión, 155 Autotransformador, 137 AWG,32,477 C

C.A., devanados de armadura, 260 capa doble, 261 ecuación de fem, 275 encordado, 261 factor de devanado, 271, 273, 275, 315 factor de distribución, 273 factor de paso, 274 paso completo, 262 paso fraccionado, 261, 274 Caída de reluctancia, 51, 75 Campo eléctrico, 35 Campo magnético, 35,51 simetna,36 Capacidades de valor nominal, 31 Caracteristica B-H, 4:¿, 46 Ciclo de histéresis, 41, 43 Ciclo de rendimiento, 31 Circuitos equivalentes, 169 Circuitos magnéticos, 53, 59 con imanes permanentes, 68 487

488

Indice

rectificador, 359 rectificador controlado en fase, 393 rectificador de silicio, 357 RSC, ver control de estado sólido, thyristor. semiconductores de potencia, 355, 357 sobreimpulso de compuerta de thyristor,388 transistor de potencia, 367 thyristor, 356,359 transistor de monounión, 390 triac,366 voltaje de pico inverso, 359 Conversión de energía, 146, 199 Corriente de excitación, 52 Curva de desmagnetización, 69, 241 D

Definición de eficiencia, 26 Definición de energía, 27 Densidad de flujo de saturación, 42 Densidad de flujo residual, 44 Deslizamiento, 267 Devanado de paso fraccional, 191 Devanado de paso completo, 191 Dimensión de estructura, 32 Dominio magnético, 41 Dualidad, 170 E

Ecuación de Bessel, 36 Ecuación de fuerza, 147, 249 Ecuación de la fuerza de Lorenz, 38 Ecuaciones de Maxwell, 37 Ecuaciones de movimiento, 159, 160 linealización, 162 solución analítica, 161 solución numérica, 166 Efecto Meisner, 40 Energía almacenada en el campo magnético, 62,65,66 Error en ángulo de fase, 138 Eslabonamiento de flujo, 61

F Factor de fuga, 72, 73, 74 Factor de inducción 52 Factor de pila, 48 Fem de movimiento 51,162,445 Fem posterior, 196 Ferritas, 40, 88 Flujo de fuga, 53 Flujo magnético, 52 Fmm, definición, 51 Fuerza coercitiva, 44

Fuerza eléctrica, 44, 145, 147, 159 Fuerza, promedio en tiempo, 156, 173

Imanes de Alnico, 69 Imán permanente, 69-77 características, 7 O conservador, 76 curva de desmagnetización, 69 factor de fuga, 72, 74 permeabilidad de rebobinado, 77 producto de energía, 69 relación de permeancia, 69 Inductancia, definición, 63 auto, 64 determinación, 435 muesca, 66 mutua, 64, 434 Inductancia de eje de cuadratura, 155, 328 Inductancia de muesca, 66 L

Laminaciones, 47,48,88 Ley de Ampere, 38,54, 55 Ley de Biot-Savart, 38 Ley de Faraday, 38, 61, 62 M

Magnetización intrínseca, 42, 241 Magnetostricción, 146 Máquina de Beckey-Robinson, 23 Máquina de conmutador de flujo, 23 Máquina de inductor, 23 Máquina de Lundell, 23 Máquina de Nadyne, 23 Máquina de rectificador rotatorio, 22 Máquina electromegnética, 20, 35 Máquina de electrostática, 20, 37 MáqUinas de c.c., 181 arranque, 245 campos de entrehierro, 210, 216 características, 207, 227-238 clasificación, 203 conmutador, 208, 213 constante de armadura, 196, 198 conversión de energía, 199 corrimiento de escobilla, 215 curva de magnetización, 197 devanado de compensación 215 devanado de solapa, 190, 191 devanado ondulado, 190, 192 devanados de armadura, 186, 190 descripción, 182 dinámica, 243,472

490

Indice

reactancia de fuga, 270, 272, 281 reactancia magnetizante, 270, 281 resistencia de rotor, 270, 272, 281 rotor devanado, 257 sin escobillas, 257 tipo jaula, 257 velocidad síncrona, 266 Motor de par de torsión, 20 N

NEMA,32 No linealidad, S8 Notación por unidad, 110 P

Par de torsión de reluctancia, 155, 311 Paso polar, 191,266 Perdida de núcleo, 47, 49, 50 aparente, 51 medición, 48 Pérdidas magnéticas, 46 Pérdida por corrientes de Eddy, 47 Pérdida por histéresis, 44, 47 pérdida aparente de núcleo, 51 Permeabilidad, 39, 42 amplitud, 42, 128 diferencial 42 incremental, 42, 77 infinita, 60 inicial, 42, pulso, 42, 77 rebobinado, 77 relativa, 39 Permeabilidad de rebobinado, 77 Permeancia, definición, 52 Potencia, definición, 26 Potencial magnético, 51 Producto de energía, 44, 69 Programas de computación, 479 Punto de equilibrio, 164 R

Reacción de inducido, 207, 320-321 Regla-Bli, 38 Regla de la mano derecha, 38, 60, 61 Relación de permeancia, 69 Reluctancia, definición, 52 Ribeteado, 53

s Saturación, 42, 48 Saturación magnética, 42, 197 Secuencia de fase, 13 2 Símbolos y unidades, 475 Sobrecargas de máquina, 31

Solenoide largo, 78 Superconductor, 40, 46 T

Tabla de conductores, 477 Temperatura de Curie, 44 Teoría del campo electromagnético. 37 Teoría general de las máquinas, 433 dinámica de la máquina, 465, 473 formulación en el dominio del tiempo, 461 inductancias de máquina, 436-444 máquina primitva, 459 técnicas de transformación, 436, 448, 452 Tipos de máquinas rotatorias, 20 Toroide, 36, 64, 66 Transformación lineal, 435 Tranformador,83 aceite, 85 acoplamiento de impedancia, 107 armónicas, 135 audio, 90 capacitancia, 122 característica de excitación, 92 circuito equivalente, 105 clasificación, 89 con carga, 101 conexiones, 126,129 conexción Scott, 136 construcción, 84 corriente, 90, 138 corriente de irrupción, 124 corriente magnetizante, 94, 96 delta abierta, 136 delta estrella, 130, 132 devanados, 84 diagrama de fasores, 94, 101, 113 ecuación de fem 95 eficiencia, 114 electrónica, 85, 90, 118 especialidad, 90 estrella de seis fases, 136 excitación de flujo senoidal, 98 excitación por corriente senoidal, 98 fmm de excitación, 102 ideal, 106, 109 impedancia, 111, 114 impedancia equivalente, 107, 111, 114 instrumento, 90 núcleo, 85, 88 pérdida por corona, 98 pérdidas de núcleo, 99, 109 polaridad, 116 potencial, 90 prueba de circuito abierto, 117, 121

Indice ecuación de fem, 195-196 ecuación de velocidad, 201 ecuaciones de regulación, 201 eficiencia, 217 entrehierro axial, 184, 194 especificaciones de placa, 221 establecimiento de voltaje, 202, 205 excitación independiente, 233, 235 fem posterior, 196 flujo de potencia, 218 frenado, 245 frenado dinámico, 246 frenado regenerativo, 246 función de transferencia, 248 imán permanente, 237-242 interpolo, 215 línea de resistencia de campo, 204 máquina en derivación, 236 motores de control, 238 motores en serie, 232 obturación, 246 par de torsión, 199 pérdidas, 217-221 plano neutral geométrico, 208 prueba de retardación, 247 pruebas, 223-227 reacción de inducido, 208 regulación de voltaje, 235 sin escobillas, 181 Sistem Ward-Leonard, 233 Máquinas de colector polifásico, 25 Máquinas de histéresis, 22, 341 Máquinas de imán permanente, 24, 238 Máquinas de inducción, 20, 257 Máquinas de reluctancia, 22, 145, 152 -155, 341 Máquinas homopolares, 20,181 Máquinas síncronas, 20, 301 ángulo de potencia, 323, 331 arranque, 313 circuitos equivalentes, 324, 334 coeficiente de fuga de campo, 332 construcción, 302 cortocircuito repentino, 332 curvas y, 324 devanado de amortiguación, 307 enfriado, 307 excitador, 302 excitador piloto, 302 fem inducida nominal, 321 impedancia síncrona, 320 motor de histéresis, 341 motor de reluctancia, 341 operación como generador 313 320 operación como motor, 3Ú, 324 par de sincronización, 337 polo saliente. 303. ,17 prueba de deslizamiento, 339 pruebas, 339

489

reacción de armadura, 320-321 reactancias, 328, 335, 339 reactancia de eje directo, 328 reactancia de eje de cuadratura, 328 reactancia de fuga, 319 reactancia síncrona, 320 reactancia subtransitoria, 334 reactancia transitoria, 334 regulación de voltaje, 321 resistencia de armadura, 320 rotor cilíndrico, 303, 320, 324 sobreexcitada, 324 subexcitado, 324 teoría de dos reacciones, 327 transitorio, 331 transitorio mecánico, 335 velocidad síncrona, 303 Material diamagnético, 40 Material ferrimagnético, 40 Material ferromagnético, 40 Material magnético, 39-46 blando, 40 diamagnético, 40 duro, 40 ferrimagnético,40 ferromagnético, 40 paramagnético, 39 Material paramagnético, 39 Material superparamagnético, 40 Motor de escalonamiento, 20 Motor de inducción, 257 acción, 266 anillo colector, 257 arranque, 288, 295 arranque con capacitor, 296 cálculos de ejecución, 276 campo magnético rotatorio, 264 circuito equivalente, 270, 279,448, 454 circuito equivalente del rotor, 268 control de velocidad, 285 criterios de ejecución, 282 desbalanceado, 457,474 deslizamiento, 267 dinámica, 465 eficiencia, 279 fase de hendedura, 295 flujo de potencia, 276 fmm de armadura, 260 frecuencia de deslizamiento, 269 jaula doble, 290 monofásico, 291-297 par de torsión, 266,279,283,456 polo sombreado, 297 potencia a través de entrehierro, 276 potencia desarrollada, 277 prueba de rotor bloqueado, 280 prueba sin carga, 279 pruebas, 279

bldlee prueba de corto-
View more...

Comments

Copyright ©2017 KUPDF Inc.
SUPPORT KUPDF